
We’ve developed the first-ever complete set of worked solutions for the IMAT 2021 exam, carefully designed to support students aiming to succeed in the IMAT 2025. These solutions follow the official 2021 exam structure and Cambridge assessment style, offering detailed, step-by-step explanations for every question. The IMAT 2021 included 22 questions in logical reasoning, problem-solving, and general knowledge, 18 in biology, 12 in chemistry, and 8 in mathematics and physics. Whether you’re reviewing past papers or building confidence for your upcoming exam, this resource is your ultimate guide to mastering the IMAT.
Correct Answer: A) Many parcels of books sent by relatives or friends have been found to contain illegal drugs.
The argument states that allowing prisoners to receive books as gifts may discourage them from working, which is important for their self-esteem and work ethic. It concludes that prisoners should only access books through the prison library instead of receiving them from relatives or friends.
To strengthen this argument, we need a statement that further justifies why preventing book parcels is beneficial—either by supporting the idea that prisoners should work or that library access is sufficient.
Option A Explanation:
A) Many parcels of books sent by relatives or friends have been found to contain illegal drugs.
Correct. This strengthens the argument by adding a security concern—if book parcels can be used to smuggle drugs, banning them becomes more reasonable.
Option B Explanation:
B) Prisoners often read detective stories which may encourage them to glamorise crime.
Irrelevant. The argument is about discouraging work, not book content.
Option C Explanation:
C) The range of books available in prison libraries is limited.
Weakens the argument. If the library selection is limited, banning book parcels reduces prisoners' access to books, making the policy less justified.
Option D Explanation:
D) Many prisoners have difficulty finding work when they are released because of their low level of literacy.
Irrelevant. The argument is about prison work ethic, not literacy levels after release.
Option E Explanation:
E) Prisoners have to purchase a range of items from their wages, including a television licence.
Irrelevant. This does not support the idea that banning book parcels encourages work.
Correct Answer: A) Positive thinking is not always helpful.
The argument states with a point that say that optimistic, happy people tend to be healthier, more physically active and more successful. Then, that point is contradicted by the third statement which says that positivity could result in negative results if not used properly. Then, examples come. So, it implies that positivity mindset might not always give positive results. We have to choose the conclusion that resembles this the most.
Option A Explanation:
A) Positive thinking is not always helpful.
This directly summarizes the argument: positivity has both benefits and downsides.
Option B Explanation:
B) Negative thinking about health can be beneficial.
Too specific. This is one example, but not the main conclusion.
Option C Explanation:
C) People who think positively live longer.
This is mentioned as a potential benefit but is not the argument's main point.
Option D Explanation:
D) Being overly positive can damage our relationships with friends.
This is one consequence but not the overall conclusion.
Option E Explanation:
E) Research into positivity has so far ignored its potential downsides.
The passage cites research showing the negative effects, so this is inaccurate.
Correct Answer: B
This is a simple visualization question. We’ll look only at the first fold.
1st fold: Left side over right → dot on center of that left face.
the paper is unfolded in the reverse sequence and is then turned over, the dot appears on the right side, centered on the right half of the paper.
Among the options, only option B satisfies this condition.
Therefore, the correct answer is option B.
A document has been prepared to include the images and diagrams related to the questions, placed directly below the worked solutions for clarity and reference.
Correct Answer: C) 11
To answer this question, we have to count the children who has more than 2 siblings.
Rajiv, Emily and Fatima each have 1 brother and 1 sister.
Candice and Marvin have 1 brother and 2 sisters.
Aruna has 2 sisters.
Phoebe has 1 brother and 3 sisters.
Leroy and Gemma have 2 brothers.
Paul has 2 brothers and 1 sister.
Callum has 3 brothers.
So, in total, 11 families have a total of 3 or more children.
Correct Answer: C) 16
Lets say that a tin of sweet has 100 g.
36% is eaten, which means 36 g is eaten. So, 64g of sweets remain.
Then, 25% of the remaining sweets is eaten.
25% of 64g would be 16g.
So, 64g - 16g = 48g of sweets remain.
The same number of sweets will be eaten on each of the following two days.
Remaining sweets = 48g - 16g - 16g = 16g which is the same as 16% because we use 100g as original content of the tin.
Thus, the correct answer is option C) 16
Correct Answer: A
To find the best value of money, compare 1 kg price of each brand.
A) ZAP : 1 kg price = 2.4€ ÷ 2 = 1.2 €
B) ZOOM : 1 kg price = 4.05€ ÷ 2 = 1.475 €
C) ZOOM : 1 kg price = 2.5 €
D) OWN BRAND : 1 kg price = 1.25 €
E) ZAP : 1 kg price = 1.30 €
So, Box of powder with the best value is A) ZAP
Correct Answer: E) Other artists make the same choice as Mondrian for the same reasons.
The passage states that Mondrian chose London over a rural area because he loved its diverse and vibrant culture. It then concludes that London has more artists than any other city in England, implying that other artists must also prefer London for similar reasons.
For the argument to hold, it assumes that other artists also choose London for the same reasons as Mondrian—this connects the premise (Mondrian’s preference) to the conclusion (London has the most artists).
Option A Explanation:
A) London is a big city because all the artists who live there boost its size and population.
Incorrect. The passage does not say that artists make London big, just that it attracts many artists.
Option B Explanation:
B) There is less inspiration for artists in rural areas.
The passage suggests that Mondrian preferred city life, but it does not state that rural areas lack inspiration.
Option C Explanation:
C) Artists can only create great works if they are happy and relaxed.
This is too extreme; the passage does not claim this as a requirement.
Option D Explanation:
D) Other big cities in England have the same attractive features as London.
The passage does not compare London to other cities in England.
Option E Explanation:
E) Other artists make the same choice as Mondrian for the same reasons.
Correct answer. The passage assumes that London has more artists because, like Mondrian, they are drawn to its diverse and vibrant culture.
Correct Answer: B
Premise: People who take their holidays in Las Vegas love to gamble.
• (If X → Y) → If someone goes to Las Vegas, they must love gambling.
Premise: Gerry is going to France for his holiday.
• (Not X) → Gerry is not going to Las Vegas.
Conclusion: Therefore, Gerry must not like gambling.
• (Not X → Not Y) → Since Gerry is not going to Las Vegas, he must not love gambling.
Option A Explanation:
A) People who eat a lot of sweets have rotten teeth. You eat a lot of sweets, so your teeth must be rotten.
• (X → Y, X is true → Y is true)
• Not a match.
Option B Explanation:
B) People who live in the city hate traffic jams. You live in the country so you must like traffic jams.
• (If X → Y, then Not X → Not Y)
• Exact parallel reasoning!
Option C Explanation:
C) You have to find the password to complete the game. You haven’t finished the game yet so you can’t have found the password.
• (If X → Y, then Not Y → Not X) (Contrapositive, which is valid reasoning)
• Not a match.
Option D Explanation:
D) You always walk when you are visiting your sister. You are not visiting your sister so that’s why you are taking the car.
• (If X → Y, then Not Y → Not X)
• Not a match.
Option E Explanation:
E) In England cars are driven on the left side of the road. We are driving on the left, so we must be in England.
• (If X → Y, then Y → X) (Affirming the consequent, another fallacy, but not the same one)
• Not a match.
Correct Answer: C) petrol
Criteria to buy a new car is to buy the cheapest car to use for one year, a total annual mileage of 20,000.
For autogas car, total fuel charge is 30 × 100 (full tank can be used for 200 miles) = 3000
Total cost = 10,000 (car cost) + 3000 = 13,000
For diesel car, total fuel charge is 50 × 40 (full tank can be used for 500 miles) = 2000
Total cost = 12,000 (car cost) + 2000 = 14,000
For petrol car, total fuel charge is 50 × 50 (full tank can be used for 200 miles) = 2500
Total cost = 10,000 (car cost) + 2500 = 12,500
For hydrogen car, total fuel charge is 50 × 100 (full tank can be used for 200 miles) = 5000
Total cost = 9,500 (car cost) + 5000 = 14,500
For electric car, total fuel charge is 5 × 100 = 500
Total cost = 12,500 (car cost) + 500 = 13,000
So, petrol car cost the lowest.
Correct Answer: C
The paragraph discusses about the driverless vehicle development and complications. Conclusion is explicitly stated in the second sentence “But we are so engrossed in the technology that we are ignoring the legal implications – and legislating for new scenarios takes time.” This is followed by the examples that point out the need for new legislation.
Thus, the conclusion that is best suited to this paragraph is
Option C) New laws are needed to determine liability for accidents involving driverless cars.
Correct Answer: B
In 1961, the Soviet Union sent the first human into space, completing an orbit around Earth.
Option A Explanation:
A) Valentina Tereshkova
First woman in space (1963), but not the first to orbit.
Option B Explanation:
B) Yuri Gagarin
Correct answer. He flew aboard Vostok 1 on April 12, 1961, becoming the first person to orbit Earth.
Option C Explanation:
C) Neil Armstrong
First person on the Moon (1969), not the first in orbit.
Option D Explanation:
D) John Glenn
First American to orbit Earth (1962), but after Gagarin.
Option E Explanation:
E) Buzz Aldrin
Second person on the Moon, not the first to orbit Earth.
Correct Answer: A
Carl Friedrich Gauss (1777–1855) was a German mathematician known for his contributions to statistics, number theory, and physics.
Option A Explanation:
A) the normal distribution curve
Correct answer. Also called the Gaussian distribution, it is a key concept in statistics and probability.
Option B Explanation:
B) the sine function
While Gauss worked on trigonometry, the sine function predates him.
Option C Explanation:
C) the law of supply and demand
An economic principle, not related to Gauss.
Option D Explanation:
D) Bayes’ theorem
Developed by Thomas Bayes, not Gauss.
Option E Explanation:
E) the uncertainty principle
Formulated by Werner Heisenberg in quantum mechanics.
Correct Answer: B
The sharing economy involves peer-to-peer access to goods/services rather than ownership (e.g., Airbnb, Uber).
Option A Explanation:
A) Unused value is wasted value.
Key principle—sharing prevents waste.
Option B Explanation:
B) Competition is dangerous.
Correct answer. The sharing economy still thrives on competition (e.g., Uber vs. Lyft).
Option C Explanation:
C) Stranger danger can be overcome.
Trust-building mechanisms (reviews, IDs) reduce risks.
Option D Explanation:
D) Access is preferable to ownership.
Central idea—why own when you can share?
Option E Explanation:
E) Transparent and open data increases innovation.
Open data fosters efficiency in sharing platforms.
Correct answer is option B) competition is dangerous.
Correct Answer: E) Federico Fellini
Amarcord (1973) is a famous Italian film known for its nostalgic and surreal style. It was directed by Federico Fellini.
Option A Explanation:
A) Stanley Kubrick
Known for 2001: A Space Odyssey, The Shining, not Amarcord.
Option B Explanation:
B) Francis Ford Coppola
Directed The Godfather series, not Amarcord.
Option C Explanation:
C) François Truffaut
A major French director, known for The 400 Blows.
Option D Explanation:
D) Alfred Hitchcock
Master of suspense, directed Psycho, Vertigo, not Amarcord.
Option E Explanation:
E) Federico Fellini
Correct answer. Fellini was an Italian director known for surreal, dreamlike films, including Amarcord.
Correct Answer:A) Bulgarian
The Romance languages evolved from Latin and include Spanish, French, Italian, Portuguese, and Romanian.
Option A Explanation:
A) Bulgarian
Correct answer. Bulgarian is a Slavic language, not Romance.
Option B Explanation:
B) Spanish
Romance language, derived from Latin.
Option C Explanation:
C) Romanian
Though geographically in Eastern Europe, it is a Romance language.
Option D Explanation:
D) Catalan
Spoken in Spain, a Romance language.
Option E Explanation:
E) French
A major Romance language.
Correct Answer: D) Euro – Hungary
Each country has an official currency
Option A Explanation:
A) Peso – Argentina
Correct. The Argentine peso (ARS) is the national currency.
Option B Explanation:
B) Baht – Thailand
Correct. The Thai baht (THB) is used in Thailand.
Option C Explanation:
C) Rand – South Africa
Correct. The South African rand (ZAR) is the official currency.
Option D Explanation:
D) Euro – Hungary
Incorrect. Hungary uses the Hungarian forint (HUF), not the Euro.
Option E Explanation:
E) Rupee – India
Correct. The Indian rupee (INR) is India’s currency.
Correct Answer: C) Mathematics
The Fields Medal is often considered the "Nobel Prize of Mathematics," awarded every four years to outstanding mathematicians under 40.
Thus, the correct option is option C) Mathematics
Correct Answer: D) Charles Dickens
The Nobel Prize in Literature is awarded to writers who have made significant contributions to literature.
Option A Explanation:
A) Pablo Neruda
Won in 1971. Chilean poet.
Option B Explanation:
B) Bob Dylan
Won in 2016. American singer-songwriter.
Option C Explanation:
C) Thomas Mann
Won in 1929. German novelist (The Magic Mountain).
Option D Explanation:
D) Charles Dickens
Did NOT win. Despite being a famous 19th-century writer (Oliver Twist, A Christmas Carol), he never received the Nobel Prize.
Option E Explanation:
E) Harold Pinter
Won in 2005. British playwright.
Correct Answer: D) Beyond Good and Evil
Primo Levi was an Italian Jewish writer known for his works on the Holocaust and survival.
Option A Explanation:
A) The Truce
Written by Levi. Sequel to “If This Is a Man”.
Option B Explanation:
B) If This Is a Man
Written by Levi. His famous memoir about Auschwitz.
Option C Explanation:
C) The Drowned and the Saved
Written by Levi. Reflections on the Holocaust.
Option D Explanation:
D) Beyond Good and Evil
Not written by Levi. This is a philosophy book by Friedrich Nietzsche.
Option E Explanation:
E) If Not Now, When?
Written by Levi. A novel about Jewish resistance fighters.
Correct Answer: C) Cape Verde – Brazil
An archipelago is a group of islands. Some belong to specific countries.
Option A Explanation:
A) Hawaii – United States of America
Correct. Hawaii is a U.S. state.
Option B Explanation:
B) Azores – Portugal
Correct. The Azores are an autonomous region of Portugal.
Option C Explanation:
C) Cape Verde – Brazil
Incorrect. Cape Verde is an independent country, not part of Brazil.
Option D Explanation:
D) Canary Islands – Spain
Correct. They are an autonomous community of Spain.
Option E Explanation:
E) Galapagos Islands – Ecuador
Correct. They belong to Ecuador.
Correct Answer: D) Senate of the Republic
In Italy, direct universal suffrage means that all eligible citizens vote directly for the representatives. The Chamber of Deputies and the Senate of the Republic are the two houses of Parliament, both elected by the people.
Option A Explanation:
A) President of the Council of Ministers
(Prime Minister) – Not directly elected by the people; appointed by the President of the Republic.
Option B Explanation:
B) High Council of the Judiciary
Not elected by the public; it consists of judges and members appointed by different institutions.
Option C Explanation:
C) Government
The government is formed by the Prime Minister and ministers, but they are not directly elected by the people.
Option D Explanation:
D) Senate of the Republic
Correct answer. Senators are elected by the Italian people through direct universal suffrage.
Option E Explanation:
E) President of the Republic
Elected by Parliament and regional representatives, not directly by the people.
Correct Answer: E) UNESCO
World Heritage Sites are places of outstanding cultural or natural importance, recognized and protected under the UNESCO World Heritage Convention.
Option A Explanation:
A) WTO
World Trade Organization – Focuses on international trade regulations and agreements, not heritage protection.
Option B Explanation:
B) WHO
World Health Organization – Concerned with global public health, not cultural or natural sites.
Option C Explanation:
C) OECD
Organisation for Economic Co-operation and Development – Works on economic policies and development, not heritage conservation.
Option D Explanation:
D) UNICEF
United Nations Children’s Fund – Focuses on children’s rights, education, and welfare, not World Heritage Sites.
Option E Explanation:
E) UNESCO
United Nations Educational, Scientific and Cultural Organization – The correct answer. It maintains the World Heritage List and works to protect and preserve cultural and natural heritage worldwide.
Correct Answer: D) 1, 2 and 3
Escherichia coli (1) contains circular DNA because it is a prokaryote (a bacterium), and prokaryotic cells have a single, circular chromosome instead of linear chromosomes.
Saccharomyces cerevisiae (yeast) (2) is a eukaryotic organism, and like other eukaryotes, its nuclear DNA is linear; however, it also contains mitochondria, which have circular DNA, so yeast cells do have some circular DNA.
Human liver cells (3) are also eukaryotic with linear nuclear DNA, but they too contain mitochondria, which have their own circular DNA. Therefore, all three — Escherichia coli, yeast, and human liver cells — contain circular DNA, though for different reasons: in E. coli it’s the main genome, and in yeast and human cells it’s mitochondrial DNA.
Correct Answer: B) row 5
The general structure of amino acid has a central carbon attached to amino group (-NH₂), carboxyl group (-COOH), Hydrogen and side chain R. In Asparagine, the side chain R has the structure -CH₂CONH₂.
Thus, image X correctly enclose the part that is common to all amino acids. Image Y correctly enclose the side chain R group of Asparagine.
The correct option is option B) row 5.
Correct Answer: C) none of them
DNA replication is the process of synthesizing DNA from DNA.
Transcription is the process of synthesizing RNA from DNA.
Analysis of the given statements
- The breaking of hydrogen bonds: Both DNA replication and transcription require the breaking of hydrogen bonds to expose the sequence for the synthesis of new DNA strands or RNA strand.
- The formation of hydrogen bonds: Complementary base pairing and formation of hydrogen bonds occur in both processes. In DNA replication, new DNA strands form hydrogen bonds with the template strand. In transcription, mRNA forms temporary hydrogen bonds with the DNA template strand before detaching.
- The formation of phosphodiester bonds: Phosphodiester bonds form in both processes to synthesize a new strand of nucleic acids. Thus, this is also a shared feature, not a distinguishing one.
Thus, none of the given information can distinguish between DNA replication and transcription.
Correct Answer: C) rhodopsin will form an opsin and retinal
Rods detect light intensity and motion in low-light conditions but not color or detail, and are mainly found at the retina’s edge.
Cones detect color (red, green, blue) and fine detail, concentrated in the center, with the fovea centralis containing only cones.
When light reaches a photoreceptor, it triggers the breakdown of rhodopsin, a molecule made of opsin and retinal. Light causes retinal to change shape and separate from opsin, initiating an action potential in the receptor cell. Retinal’s structure is similar to Vitamin A, and a deficiency in Vitamin A can lead to vision problems, particularly in low-light conditions.
Correct Answer: D) 3
The diagram represents the fluid-mosaic model of a cell membrane.
Label 1 indicates the carbohydrate molecule attaching to the external surface of the cell membrane. It is a polar molecule
Label 2 indicates the phospholipid head, which is hydrophilic and faces outside.
Label 3 indicates the phospholipid tail, which is made up of hydrophobic fatty acid chains and face inside. This shows the hydrophobic region of a molecule.
Label 4 indicates a part of the membrane protein that is facing outside in exposure to water. This is hydrophilic.
Label 5 indicates a part of the peripheral protein that faces the outside in exposure to water. This is another hydrophilic part.
Thus, the correct option is option D) 3.
Correct Answer: E
A cell from an organism with a haploid number of 4 have 8 homologous chromosomes.
Option A Explanation:
Diagram does not have a correct description because in Meiosis I, homologous chromosomes separate, not sister chromatids.
Option B Explanation:
Diagram does not have a correct description because in Mitosis, all 8 homologous chromosomes align in metaphase plate during mitosis and then separate in Anaphase. The diagram mentions only 4 chromosomes.
Option C Explanation:
Diagram does not have a correct description because in Mitosis, sister chromatids separate, not homologous chromosomes.
Option D Explanation:
Diagram does not have a correct description because in Mitosis, all 8 homologous chromosomes align in metaphase plate during metaphase, not 4.
Option E Explanation:
Diagram has a correct description. The cell is in prophase, there are 8 chromosomes, in condensed shape without mitotic spindles.
Correct Answer: B) 2 only
Speciation
There are 2 main types of speciation:
- Allopatric Speciation (“Geographical Isolation”)
- Sympatric Speciation (“No Physical Barrier”, A new species evolves within the same geographical area) – This is not the case here because the two parents are from different species and different region. Therefore, the first term illustrated in the question is incorrect.
Isolation
Isolation prevents gene flow between populations, allowing genetic differences to accumulate. In this case, the new species no longer mates with the original parent generation, so reproductive isolation is happening. (The 2nd term illustrated in the question is correct).
There are two main types of reproductive isolation that contribute to speciation:
- Pre-zygotic Isolation (Before Fertilization)
- Post-zygotic Isolation (After Fertilization)
In this case, the success of the new species is due to the pre-zygotic isolation because the reproductive barrier here is behavioral (pre-zygotic) — through song differences and large bill and body size, leading to mate choice preferences before fertilization.
(Mating doesn’t happen with original parent generation). So, it is not post-zygotic. (The 3rd term illustrated in question is wrong)
Thus, the only correct term for the success of new species is: Reproductive isolation.
Correct Answer: B) 1, 2, and 3
A single base mutation (point mutation) can have different effects on the protein, depending on the type of mutation that occurs.
These are
-
Missense mutation is a type of point mutation where a single base change in the DNA sequence leads to the substitution of one amino acid for another in the protein.
This can result in a change in the protein’s tertiary structure, potentially altering its function. - Nonsense mutation is a type of point mutation where a single base change converts a codon into a premature stop codon, causing early termination of translation and producing a shorter, usually nonfunctional protein.
- Silent mutation is a type of point mutation where a single base change occurs but does not alter the amino acid sequence of the protein, due to the redundancy (degeneracy) of the genetic code.
Thus, all of the mentioned effects are the possible effects on a single base mutation.
Correct Answer: C) oxidation of NADH
Catabolism refers to the breakdown of molecules to release energy. It is the destructive phase of metabolism, in contrast to anabolism, which involves building complex molecules.
Option A Explanation:
A) formation of RuBisCo
Anabolic
RuBisCo (Ribulose-1,5-bisphosphate carboxylase/oxygenase) is an enzyme involved in the Calvin cycle (photosynthesis), which is an anabolic process. Humans do not synthesize RuBisCo.
Option B Explanation:
B) replication of DNA
Anabolic
DNA replication builds new DNA strands, requiring nucleotides and ATP, making it an anabolic process.
Option C Explanation:
C) oxidation of NADH
Catabolic
NADH is oxidized to NAD⁺ in the electron transport chain (ETC) during cellular respiration. This process releases energy to drive ATP production, making it a classic example of catabolism.
Option D Explanation:
D) ATP made during glycolysis
ATP synthesis itself is not catabolic; rather, it is a result of catabolic reactions (like glycolysis).
Option E Explanation:
E) attachment of tRNA to mRNA
Anabolic
This occurs during translation, which is an anabolic process because it builds proteins from amino acids.
Correct Answer: D) 1 and 2 only
Analysis of the given functions
1) They stimulate cell division in cotton plant seeds.
• Agrobacterium tumefaciens is known for causing tumors in plants by inserting its tumor-inducing plasmid into plant cells.
• This stimulates uncontrolled cell division, leading to tumor formation, which is used here to grow genetically modified cotton plants.
2) They supply the plasmid vector.
• The plasmid from Agrobacterium tumefaciens is used as a vector to transfer the insect toxin gene (from Bacillus thuringiensis) into cotton plant cells.
3) They are the source of the gene for the production of the insect toxin.
• Incorrect because the insect toxin gene comes from Bacillus thuringiensis, not Agrobacterium tumefaciens.
4) They are the source of the gene for antibiotic resistance.
• Incorrect because the antibiotic resistance gene is artificially inserted into the plasmid, not naturally provided by Agrobacterium tumefaciens.
Thus, TWO correct functions of the Agrobacterium tumefaciens bacteria in this process are 1 and 2.
Correct Answer: A) Blood entering the right atrium can bypass the lungs and go to the rest of the body.
In a developing human fetus, the lungs are non-functional because the fetus gets oxygen from the placenta through the umbilical cord. To bypass the lungs, the fetal circulation has two key shunts:
- Foramen ovale – A hole in the atrial septum that allows blood to flow from the right atrium to the left atrium, bypassing the lungs.
- Ductus arteriosus – A vessel that connects the pulmonary artery to the aorta, further bypassing the lungs.
Since the foramen ovale is an opening in the septum, blood from the right atrium can directly enter the left atrium, skipping the pulmonary circulation and being pumped to the rest of the body.
Option A Explanation:
A) Blood entering the right atrium can bypass the lungs and go to the rest of the body.
This statement is correct.
Option B Explanation:
B) Blood in the right atrium may return directly to the vena cava.
Blood does not flow backward into the vena cava; it moves forward into the left atrium through the foramen ovale.
Option C Explanation:
C) The atrioventricular node will not cause ventricular systole.
The atrioventricular (AV) node remains functional, and ventricular systole still occurs normally, even in a fetus.
Option D Explanation:
D) Blood in the pulmonary artery will go directly to the aorta.
This describes the function of the ductus arteriosus, not the hole in the septum (foramen ovale).
Option E Explanation:
E) The maximum pressure in the aorta is the same as the maximum pressure in the contracting atrium.
The aortic pressure is much higher than atrial pressure because the aorta supplies systemic circulation, which has high resistance.
Correct Answer: B) 1, 2, and 4 only
Bile is a digestive fluid produced by the liver and stored in the gallbladder. It plays a crucial role in fat digestion by emulsifying large lipid droplets into smaller ones, increasing their surface area for enzymes like lipase to act on, which enhances fat breakdown and absorption in the small intestine.
Analysis of the given statements
-
It emulsifies fat globules.
Bile contains bile salts, which break large fat globules into smaller droplets (emulsification). This increases the surface area for lipase to act on, aiding in fat digestion. Fat globules are large, spherical droplets of fat that form when lipids are released into the digestive system. They are hydrophobic and tend to clump together in water-based environments like the small intestine. -
It dilutes the contents of the small intestine.
When released into the duodenum, it dilutes chyme (partially digested food) and facilitates digestion. -
It lowers the activation energy of lipase.
Bile does not function as an enzyme. -
It helps to reduce the acidity of the contents leaving the stomach.
Bile contains bicarbonate (HCO₃⁻), which neutralizes acidic chyme from the stomach.
Thus, the correct statements are 1, 2 and 4.
Correct Answer: B) 2, 3, 1
Immediately after acetylcholine binds to postsynaptic receptors of a muscle fibre, Na+ channels open, leading to depolarization of the membrane and the generation of an action potential. The action potential travels along the T-tubules and reaches the sarcoplasmic reticulum (SR). This causes voltage-gated Ca²⁺ channels in the SR to open, releasing Ca²⁺ ions into the sarcoplasm. Ca²⁺ binds to troponin, which causes a conformational change. This moves tropomyosin away from the myosin-binding sites on the actin filament, allowing myosin to bind to actin and initiate contraction.
Thus, the correct order is 2,3,1
Correct Answer: E) 1 only
Features of HIV
- HIV (Human Immunodeficiency Virus) is an enveloped virus, and like all viruses, it has a protein capsid that encloses its genetic material.
- It is a retrovirus, meaning its genetic material is single-stranded RNA (ssRNA), not DNA. HIV carries three essential enzymes: reverse transcriptase, integrase and protease.
- Viruses do not have ribosomes. They rely entirely on the host cell’s ribosomes for protein synthesis because they cannot make proteins on their own.
Thus, the correct option is option E) 1 only.
Correct Answer: D) glomerulus
Option A Explanation:
A) distal convoluted tubule
Reabsorbs water under the influence of aldosterone and ADH and plays a role in fine-tuning ion and fluid balance. Remember, ADH has effect on both DCT and collecting duct, but it has an even stronger effect, allowing much more water reabsorption depending on body hydration.
Option B Explanation:
B) collecting duct
Regulates final water reabsorption under the influence of antidiuretic hormone (ADH), concentrating urine when necessary.
Option C Explanation:
C) proximal convoluted tubule
Reabsorbs about 65–70% of water by passive transport, along with glucose, amino acids, and ions, through active and passive transport mechanisms.
Option D Explanation:
D) glomerulus
The glomerulus is responsible for filtration, not reabsorption.
Option E Explanation:
E) loop of Henle
Specifically, the descending limb is permeable to water, allowing further water reabsorption via osmosis, while the ascending limb is impermeable to water.
Correct Answer: A) row 1
The Y chromosome consists of tightly packed nucleosomes, which are fundamental units of chromatin. Each nucleosome is composed of a core of eight histone proteins, around which DNA is wrapped twice, ensuring efficient packaging and regulation of genetic material. Thus, the structure should be ordered in Y chromosome → nucleosome → histone (from largest to smallest).
X chromosome is bigger than Y chromosome.
Thus, the correct option would be option A) row 1
Correct Answer: B
Living organisms are defined by several fundamental characteristics. They are composed of one or more cells, the basic units of life. They perform metabolism, carrying out chemical reactions to obtain and use energy. Living things maintain homeostasis, keeping internal conditions stable. They undergo growth and development, increasing in size and complexity. Reproduction ensures the continuation of their species, either sexually or asexually. Organisms also respond to stimuli from their environment and exhibit heredity, passing genetic information (typically DNA) to their offspring. Together, these features distinguish living organisms from non-living matter.
Analysis of each statement
-
They are made up of one or more cells.
This is a fundamental principle of cell theory: all living organisms consist of one or more cells. Unicellular organisms (like bacteria) have a single cell, while multicellular organisms (like humans) have many. -
They have DNA as the genetic material.
DNA (deoxyribonucleic acid) is the universal genetic material in known living organisms. -
They use energy to stay alive when their surroundings change.
Organisms require energy (from food, sunlight, or chemical sources) to maintain homeostasis and survive environmental changes. -
They undergo phases of growth.
Growth is a characteristic of all living organisms. Cells divide and increase in size, allowing unicellular organisms to expand and multicellular organisms to develop and mature.
Correct Answer: E) 2
Diagram 1 describes a chloroplast. The size is
(4 × 10-2) / 4000 = 1 × 10-5 m
Diagram 2 describes a bacteria. The size is 0.5 micrometer which is
5 × 10-7 m
Diagram 3 describes a mitochondria. The size is
(3 × 10-2) / 10000 = 3 × 10-6 m
Diagram 4 describes possibly a cell. The size is 0.14 mm which is
1.4 × 10-4 m
Diagram 5 describes a neuron. The size is
(3 × 10-2) / 4000 = 7.5 × 10-6 m
Comparing all the structures, the smallest one is bacteria.
Thus, the correct option is option E) 2.
Correct Answer: C
Br2 consists of two bromine (Br) atoms covalently bonded.
Bromine is a diatomic element, meaning it naturally exists as a molecule of two atoms.
Br2 is not a compound because it contains only one type of element (bromine).
Option A Explanation:
A) a compound of the element bromine
A compound consists of two or more different elements. Br2 contains only bromine, so it is not a compound.
Option B Explanation:
B) two separate atoms of the element bromine
Br2 is a single molecule where two bromine atoms are covalently bonded. It does not represent two separate bromine atoms.
Option C Explanation:
C) a molecule of the element bromine
Br2 is a molecule because it consists of two atoms chemically bonded. Since it contains only bromine atoms, it is a molecule of the element bromine.
Option D Explanation:
D) a molecule of the compound bromine
Br2 is not a compound because it contains only one type of element. A compound must have two or more different elements.
Option E Explanation:
E) two bromide ions
Br2 is neutral, not charged. Bromide ions (Br−) are negatively charged and exist as separate entities in ionic compounds like NaBr.
Correct Answer: D) x + 6
Atomic number (Z) = $x$
Mass number (A) = $2x+6$
Number of neutrons = A - Z = (2x+6)-x=x+6
Correct Answer: A) Mg + 2H₂O → Mg(OH)₂ + H₂
As a general rule, if a metal reacts with cold water, the metal hydroxide and hydrogen is produced. If it reacts with steam, the metal oxide and hydrogen is formed. This is because the metal hydroxides thermally decompose to the oxide and water.
Option A Explanation:
A) Mg + 2H2O → Mg(OH)2 + H2
Reaction of Magnesium with cold water produces magnesium hydroxide (Mg(OH)2) and Hydrogen gas. This equation correctly describes the reaction happened and it is balanced.
Option B Explanation:
B) Mg + H2O → Mg(OH)2 + H2
The equation is not balanced.
Option C Explanation:
C) 2Mg + 2H2O → 2MgOH + H2
MgOH is not a valid compound. MgOH would have an unbalanced charge (+2 from Mg and –1 from OH), so it's chemically invalid.
Option D Explanation:
D) Mg + H2O → MgO + H2
Magnesium reacts very slowly with cold water, producing only small amounts of magnesium hydroxide and hydrogen gas, a reaction often considered negligible. However, when exposed to steam, magnesium reacts much more readily, forming magnesium oxide (MgO) and hydrogen gas. The reaction with steam is faster and more complete, favoring the formation of the oxide rather than the hydroxide, which is typical in the slower reaction with cold water.
Option E Explanation:
E) 2Mg + H2O → MgO + H2
Wrong stoichiometry and product
Correct Answer: B) CH₄
Intermolecular forces (IMFs) are the attractive forces between molecules. They determine properties like boiling point, solubility, and state of matter.
The main types of intermolecular forces are:
- London dispersion forces (weakest) among all types of molecules, but the only type of intermolecular forces between the non-polar molecules.
- Dipole-dipole interactions, between the polar molecules.
- Hydrogen bonding (strongest), between the polar molecules which are able to donate and accept hydrogen bonds.
Option A Explanation:
A) H2O (Water)
- Highly polar molecule.
- Forms hydrogen bonds (H attached to O).
Option B Explanation:
B) CH4 (Methane)
- Nonpolar molecule (tetrahedral symmetry cancels dipoles).
- Only London Dispersion Forces.
Option C Explanation:
C) HF (Hydrogen Fluoride)
- Very polar molecule.
- Strong hydrogen bonds (H attached to F).
Option D Explanation:
D) NH3 (Ammonia)
- Polar molecule.
- Forms hydrogen bonds (H attached to N).
Option E Explanation:
E) CO2 (Carbon Dioxide)
- Nonpolar (linear symmetry cancels dipoles).
- Only London Dispersion Forces but more electrons than CH4 → slightly stronger IMFs than CH4.
The molecule with the weakest overall intermolecular forces is CH4 (methane) because:
- It is nonpolar and it only has weak London Dispersion Forces.
Thus, the correct option is option B) CH4.
Correct Answer: E) 1 and 4 only
Sucrose, C12H22O11, is added to water at 20 °C until no more sucrose will dissolve.
Analysis of the given statements
1. A saturated solution of sucrose is produced. (Correct)
A solution is saturated when no more solute (sucrose) can dissolve at a given temperature. Since the problem states that sucrose is added until no more dissolves, a saturated solution is formed.
2. The freezing point of the solution is 0 °C. (Incorrect)
When a solute (sucrose) is dissolved in water, the freezing point decreases due to freezing point depression. The solution will freeze at a temperature lower than 0 °C, depending on the concentration of sucrose.
3. Sucrose is a non-polar substance. (Incorrect)
Sucrose is polar due to its numerous OH (hydroxyl) groups, which allow it to dissolve in water (a polar solvent). If sucrose were non-polar, it wouldn’t dissolve in water.
4. Sucrose is the solute. (Correct)
In a solution, the solute is the substance that dissolves, and the solvent is the substance that does the dissolving. Here, sucrose is dissolved in water, making it the solute.
Thus, the correct option is option E) 1 and 4 only.
Correct Answer: A) 1 and 3 only
The first pair of molecules are cyclopentane and pentene.
- Cyclopentane has the molecular formula C5H10.
- Pentene refers to a group of alkenes with the molecular formula C5H10.
Since they have the same molecular formula but different connectivity, they are structural (constitutional) isomers.
The second pair of molecules are 3-chloro-3-methylheptane and 3-chloro-3-methylheptane.
- Both molecules have the same molecular formula (C8H17Cl) and the same connectivity of atoms.
However, they differ in spatial arrangement (e.g., due to chirality), they are stereoisomers, not structural isomers.
The third pair of molecules are butanenitrile and 2-buten-1-amine.
- Both molecules have the same molecular formula (C4H7N)
- One has a nitrile (–CN) functional group. The other has an amine (–NH2) functional group and a C=C double bond.
Since they have the same molecular formula but different connectivity, they are structural (constitutional) isomers.
Thus, the correct option is option A) 1 and 3 only.
Correct Answer: C) a decrease in temperature
Equilibrium Constant Expression:
Kc = [P]2 / ([Q][R])
ΔH is negative, meaning the reaction is exothermic.
We need to determine which factor increases Kc.
Option A Explanation:
A) an increase in pressure
Pressure affects equilibrium position but not the equilibrium constant Kc.
Option B Explanation:
B) use of a suitable catalyst
A catalyst only speeds up reaction rates without affecting the position of equilibrium or Kc.
Option C Explanation:
C) a decrease in temperature
Since the reaction is exothermic, decreasing the temperature shifts equilibrium towards the products (favoring P), increasing Kc.
This option is correct.
Option D Explanation:
D) a decrease in pressure
Pressure affects equilibrium position but not the equilibrium constant Kc.
Option E Explanation:
E) an increase in temperature
If temperature increases, the equilibrium shifts towards the reactants (favoring Q and R), decreasing Kc.
Correct Answer: A) NF₃
To determine which molecule has a permanent overall dipole moment, we need to analyze molecular geometry and bond dipoles. A molecule has a permanent dipole moment if:
- It has polar bonds (atoms with different electronegativities).
- The geometry does not cancel out the dipoles.
Option A Explanation:
A) NF3
Shape: Trigonal pyramidal (like NH3)
Bond Polarity: N–F bonds are polar (F is more electronegative than N).
Dipole Moment: The dipoles do not cancel due to the lone pair on N, leading to an asymmetric charge distribution. NF3 has a permanent dipole moment.
Option B Explanation:
B) PCl5
Shape: Trigonal bipyramidal.
Bond Polarity: P–Cl bonds are polar.
Dipole Moment: The molecular symmetry cancels out the dipoles. No overall dipole moment.
Option C Explanation:
C) SF6
Shape: Octahedral.
Bond Polarity: S–F bonds are polar.
Dipole Moment: The symmetry causes the dipoles to cancel. No overall dipole moment.
Option D Explanation:
D) CCl4
Shape: Tetrahedral.
Bond Polarity: C–Cl bonds are polar.
Dipole Moment: The symmetry cancels the dipoles. No overall dipole moment.
Option E Explanation:
E) CO2
Shape: Linear.
Bond Polarity: C=O bonds are polar.
Dipole Moment: The two bond dipoles are equal and opposite, canceling out. No overall dipole moment.
Correct Answer: A) thiol
In the given cysteine structure, carboxyl group (-COOH) and amino group (-NH) can be identified on each end of the molecule. Thiol group (-SH) can be identified as side chain (R) of this amino acid; cysteine.
Correct Answer: E) V₂O₅ and VO₂⁺
To determine which pair of species contains vanadium (V) in the same oxidation state, we need to calculate its oxidation number in each compound.
Option A Explanation:
A) VO2 and VO3−
Let the oxidation state of V be x.
For VO2:
x + 2(–2) = 0
x – 4 = 0
x = +4
For VO3−:
x + 3(–2) = –1
x – 6 = –1
x = +5
Different oxidation states.
Option B Explanation:
B) VO2 and V2O5
For VO2: (calculated above) → x = +4
For V2O5:
2x + 5(–2) = 0
2x – 10 = 0
x = +5
Different oxidation states.
Option C Explanation:
C) VCl4 and VO3−
For VCl4:
x + 4(–1) = 0
x = +4
For VO3−: (calculated above) → x = +5
Different oxidation states.
Option D Explanation:
D) VCl4 and VO43−
For VCl4: (calculated above) → x = +4
For VO43−:
x + 4(–2) = –3
x – 8 = –3
x = +5
Different oxidation states.
Option E Explanation:
E) V2O5 and VO3−
For V2O5: (calculated above) → x = +5
For VO3−: (calculated above) → x = +5
Same oxidation state.
Correct Answer: B) X has a hydrogen ion concentration 100 times greater than Y.
pH = –log [H+]
X: pH = 2, [H+] = 10–2
Y: pH = 4, [H+] = 10–4
Comparing the two solutions, H+ concentration of X is 100 times greater than that of Y.
Correct Answer: A) 20 g L⁻¹
Given:
Volume of NaOH = 40 cm³
Concentration of H2SO4 = 25 cm³ = 0.025 L (1 L = 1000 cm³)
Volume of H2SO4 = 0.40 mol L–1
To find the concentration of NaOH, we first need to find the mole of H2SO4 reacted.
mole of H2SO4 = concentration × volume = 0.40 mol L–1 × 0.025 L = 0.01 mol
From the balanced equation, 2 moles of NaOH react with 1 mole of H2SO4.
moles of NaOH = 2 × 0.01 = 0.02 mol
Thus, concentration of NaOH = moles / volume = 0.02 mol / 0.04 L = 0.5 mol L–1
To convert the unit mol L–1 to g L–1:
We are given the molar mass of NaOH: Mr of NaOH = 40 g/mol
Concentration in g/L = 0.5 × 40 = 20 g/L
We have to eliminate the square root from the denominator (rationalization). So, let's look at the steps to solve this problem.
Step 1: Rationalize the Denominator
Multiply the numerator and denominator by the conjugate of the denominator. The conjugate of 2 + √3 is 2 − √3 :
2 / (2 − √3) × (2 − √3) / (2 − √3) = 2(2 − √3) / ((2 + √3)(2 − √3))
Step 2: Expand the Numerator and Denominator
Numerator: 2(2 − √3) = 4 − 2√3
Denominator (difference of squares): (2 + √3)(2 − √3) = (2)² − (√3)² = 4 − 3 = 1
Step 3: Simplify the expression:
So, the expression simplifies to:
2 / (2 − √3) = (4 − 2√3) / 1 = 4 − 2√3
We will use the vertex formula for a quadratic equation y = ax² + bx + c. Let's look at the steps to solve this question.
Step 1: Identify Coefficients of the Quadratic Equation
Comparing the given quadratic equation y = 2x² + 5x − 2 with the general form and we have the coefficients: a = 2, b = 5, c = −2
Step 2: Calculate the Discriminant D
The discriminant D is given by:
Step 3: Use the Formula
The formula to find the x and y coordinates is:
(x, y) = (−5 / 2×2, −41 / 4×2)
(x, y) = (−5 / 4, −41 / 8)
We have to find the initial vertical acceleration of the rocket. We will apply Newton's second law to the forces acting on the rocket and find the net force.
As the rocket moves upwards, hence the magnitude of thrust is greater than the downward weight.
So, Net force: Fnet = T − mg
Now, acceleration is net force upon mass, so:
We have to simplify the expression and find the value of x:
82x+3 × (1⁄4)3x = 2x+3
So, first express all terms with the same base (2) and then solve for x.
Let's do this step by step.
Step 1: Rewrite Bases
8 = 23, 4 = 22
Substitute:
(23)2x+3 × (22)−3x = 2x+3
Step 2: Simplify Exponents
26x + 9 × 2−6x = 2x + 3
26x + 9 − 6x = 2x + 3
29 = 2x + 3
Step 3: Equate Exponents
Since the bases are equal:
9 = x + 3
⇒ x = 6
Given the mean meals for male and female students and we have to find the mean for females (x).
Let's solve step by step.
Step 1: Calculate Total Students and Meals
Total students: 39 + 36 = 75
Combined mean: 3 1⁄5 = 16⁄5
Total meals: 75 × 16⁄5 = 240
Step 2: Calculate Meals for Males
Mean for males: 2
Total meals for males: 39 × 2 = 78
Step 3: Solve for Females
Total meals for females: 240 − 78 = 162
Mean for females: 162⁄36 = 4.5 = 4 1⁄2
Power is the rate of energy transfer. It is given by the energy supplied over the time taken.
P = Q⁄t
So, the total Energy Supplied by the Heater is:
Q = P · t = 100W × 16000s = 160000 J
Specific Latent Heat of Evaporation is the energy required to change 1 gram of a substance from liquid to gas without a temperature change (at the boiling point).
Q = m · L
Where:
→ Q = Energy supplied (Joules),
→ m = Mass vaporized (grams),
→ L = Specific latent heat (J/g)
The energy Q is used to vaporize 200 g of the liquid. Using the latent heat formula:
Q = m · L ⇒ 160,000 J = 200 g × L
So, Specific Latent Heat (L) is:
L = 160,000⁄200 J/g = 800 J/g
Let’s look at each of the statements and analyse them:
-
Statement 1: "Incident rays travelling parallel to the principal axis always pass through F after reflection." – True.
This is a fundamental property of concave mirrors (parabolic or spherical). Rays parallel to the principal axis converge at the focus (F) after reflection. -
Statement 2: "Incident rays passing through F always travel parallel to the principal axis after reflection." – True.
This is the reverse of Statement 1 and is also true due to the reversibility of light paths in mirrors. - Statement 3: "The image formed is always inverted." – False. For a concave mirror, the image is inverted only when the object is placed beyond the focal point (F).
- Statement 4: "The image formed is always real." – False. The image is real only when the object is placed beyond the focal point.
-
Statement 5: "The image formed is always larger than the object." – False. The size of the image depends on the object's position.
For example, if the object is very far away, the image is smaller. If the object is close to the mirror, the image can be larger or smaller.
Hence, Statements 1 & 2 are correct.
It’s given that the boat’s speed in still water (relative to water) is 4 m/s,
and the flow speed of the river is 3 m/s (parallel to the banks).
So, the resultant velocity of the boat is the vector sum of these two velocities:
vresultant = √(4² + 3²) = √(16 + 9) = √25 = 5 m/s
Vector diagram:
- 4 m/s upward (Boat’s speed)
- 3 m/s to the right (River flow)
- Resultant: 5 m/s (hypotenuse)
Now, the angle is measured from the line perpendicular to the bank (90° line).
The options are given in terms of cos⁻¹.
cos(θ) = adjacent ⁄ hypotenuse = 4 ⁄ 5
⇒ θ = cos⁻¹(4⁄5)
A document has been prepared to include the images and diagrams related to the questions, placed directly below the worked solutions for clarity and reference.{"type":"elementor","siteurl":"https://thisislocomotive.com/wp-json/","elements":[{"id":"0412637","elType":"container","isInner":false,"isLocked":false,"settings":{"hover_parallax":[{"layer_position_vr":{"unit":"%","size":30},"layer_position_hr":{"unit":"%","size":40},"_id":"8df92da","repeater_bg_image":{"url":"https://thisislocomotive.com/wp-content/plugins/elementor/assets/images/placeholder.png","id":"","size":""},"layer_width":100,"layer_position_hr_tablet":{"unit":"px","size":"","sizes":[]},"layer_position_hr_mobile_extra":{"unit":"px","size":"","sizes":[]},"layer_position_hr_mobile":{"unit":"px","size":"","sizes":[]},"layer_position_vr_tablet":{"unit":"px","size":"","sizes":[]},"layer_position_vr_mobile_extra":{"unit":"px","size":"","sizes":[]},"layer_position_vr_mobile":{"unit":"px","size":"","sizes":[]},"data_depth":0.4},{"layer_position_vr":{"unit":"%","size":60},"layer_position_hr":{"unit":"%","size":20},"_id":"16bc36f","repeater_bg_image":{"url":"https://thisislocomotive.com/wp-content/plugins/elementor/assets/images/placeholder.png","id":"","size":""},"layer_width":100,"layer_position_hr_tablet":{"unit":"px","size":"","sizes":[]},"layer_position_hr_mobile_extra":{"unit":"px","size":"","sizes":[]},"layer_position_hr_mobile":{"unit":"px","size":"","sizes":[]},"layer_position_vr_tablet":{"unit":"px","size":"","sizes":[]},"layer_position_vr_mobile_extra":{"unit":"px","size":"","sizes":[]},"layer_position_vr_mobile":{"unit":"px","size":"","sizes":[]},"data_depth":0.4}],"container_type":"flex","content_width":"boxed","width":{"unit":"%","size":"","sizes":[]},"width_tablet":{"unit":"px","size":"","sizes":[]},"width_mobile_extra":{"unit":"px","size":"","sizes":[]},"width_mobile":{"unit":"px","size":"","sizes":[]},"boxed_width":{"unit":"px","size":"","sizes":[]},"boxed_width_tablet":{"unit":"px","size":"","sizes":[]},"boxed_width_mobile_extra":{"unit":"px","size":"","sizes":[]},"boxed_width_mobile":{"unit":"px","size":"","sizes":[]},"min_height":{"unit":"px","size":"","sizes":[]},"min_height_tablet":{"unit":"px","size":"","sizes":[]},"min_height_mobile_extra":{"unit":"px","size":"","sizes":[]},"min_height_mobile":{"unit":"px","size":"","sizes":[]},"flex_direction":"","flex_direction_tablet":"","flex_direction_mobile_extra":"","flex_direction_mobile":"","flex__is_row":"row","flex__is_column":"column","flex_justify_content":"","flex_justify_content_tablet":"","flex_justify_content_mobile_extra":"","flex_justify_content_mobile":"","flex_align_items":"","flex_align_items_tablet":"","flex_align_items_mobile_extra":"","flex_align_items_mobile":"","flex_gap":{"column":"","row":"","isLinked":true,"unit":"px"},"flex_gap_tablet":{"column":"","row":"","isLinked":true,"unit":"px"},"flex_gap_mobile_extra":{"column":"","row":"","isLinked":true,"unit":"px"},"flex_gap_mobile":{"column":"","row":"","isLinked":true,"unit":"px"},"flex_wrap":"","flex_wrap_tablet":"","flex_wrap_mobile_extra":"","flex_wrap_mobile":"","flex_align_content":"","flex_align_content_tablet":"","flex_align_content_mobile_extra":"","flex_align_content_mobile":"","grid_outline":"yes","grid_columns_grid":{"unit":"fr","size":3,"sizes":[]},"grid_columns_grid_tablet":{"unit":"fr","size":"","sizes":[]},"grid_columns_grid_mobile_extra":{"unit":"fr","size":"","sizes":[]},"grid_columns_grid_mobile":{"unit":"fr","size":1,"sizes":[]},"grid_rows_grid":{"unit":"fr","size":2,"sizes":[]},"grid_rows_grid_tablet":{"unit":"fr","size":"","sizes":[]},"grid_rows_grid_mobile_extra":{"unit":"fr","size":"","sizes":[]},"grid_rows_grid_mobile":{"unit":"fr","size":"","sizes":[]},"grid_gaps":{"column":"","row":"","isLinked":true,"unit":"px"},"grid_gaps_tablet":{"column":"","row":"","isLinked":true,"unit":"px"},"grid_gaps_mobile_extra":{"column":"","row":"","isLinked":true,"unit":"px"},"grid_gaps_mobile":{"column":"","row":"","isLinked":true,"unit":"px"},"grid_auto_flow":"row","grid_auto_flow_tablet":"row","grid_auto_flow_mobile_extra":"row","grid_auto_flow_mobile":"row","grid_justify_items":"","grid_justify_items_tablet":"","grid_justify_items_mobile_extra":"","grid_justify_items_mobile":"","grid_align_items":"","grid_align_items_tablet":"","grid_align_items_mobile_extra":"","grid_align_items_mobile":"","grid_justify_content":"","grid_justify_content_tablet":"","grid_justify_content_mobile_extra":"","grid_justify_content_mobile":"","grid_align_content":"","grid_align_content_tablet":"","grid_align_content_mobile_extra":"","grid_align_content_mobile":"","grid__is_row":"row","grid__is_column":"column","overflow":"","html_tag":"","link":{"url":"","is_external":"","nofollow":"","custom_attributes":""},"background_background":"","background_color":"","background_color_stop":{"unit":"%","size":0,"sizes":[]},"background_color_stop_tablet":{"unit":"%"},"background_color_stop_mobile_extra":{"unit":"%"},"background_color_stop_mobile":{"unit":"%"},"background_color_b":"#f2295b","background_color_b_stop":{"unit":"%","size":100,"sizes":[]},"background_color_b_stop_tablet":{"unit":"%"},"background_color_b_stop_mobile_extra":{"unit":"%"},"background_color_b_stop_mobile":{"unit":"%"},"background_gradient_type":"linear","background_gradient_angle":{"unit":"deg","size":180,"sizes":[]},"background_gradient_angle_tablet":{"unit":"deg"},"background_gradient_angle_mobile_extra":{"unit":"deg"},"background_gradient_angle_mobile":{"unit":"deg"},"background_gradient_position":"center center","background_gradient_position_tablet":"","background_gradient_position_mobile_extra":"","background_gradient_position_mobile":"","background_image":{"url":"","id":"","size":""},"background_image_tablet":{"url":"","id":"","size":""},"background_image_mobile_extra":{"url":"","id":"","size":""},"background_image_mobile":{"url":"","id":"","size":""},"background_position":"","background_position_tablet":"","background_position_mobile_extra":"","background_position_mobile":"","background_xpos":{"unit":"px","size":0,"sizes":[]},"background_xpos_tablet":{"unit":"px","size":0,"sizes":[]},"background_xpos_mobile_extra":{"unit":"px","size":"","sizes":[]},"background_xpos_mobile":{"unit":"px","size":0,"sizes":[]},"background_ypos":{"unit":"px","size":0,"sizes":[]},"background_ypos_tablet":{"unit":"px","size":0,"sizes":[]},"background_ypos_mobile_extra":{"unit":"px","size":"","sizes":[]},"background_ypos_mobile":{"unit":"px","size":0,"sizes":[]},"background_attachment":"","background_repeat":"","background_repeat_tablet":"","background_repeat_mobile_extra":"","background_repeat_mobile":"","background_size":"","background_size_tablet":"","background_size_mobile_extra":"","background_size_mobile":"","background_bg_width":{"unit":"%","size":100,"sizes":[]},"background_bg_width_tablet":{"unit":"px","size":"","sizes":[]},"background_bg_width_mobile_extra":{"unit":"px","size":"","sizes":[]},"background_bg_width_mobile":{"unit":"px","size":"","sizes":[]},"background_video_link":"","background_video_start":"","background_video_end":"","background_play_once":"","background_play_on_mobile":"","background_privacy_mode":"","background_video_fallback":{"url":"","id":"","size":""},"background_slideshow_gallery":[],"background_slideshow_loop":"yes","background_slideshow_slide_duration":5000,"background_slideshow_slide_transition":"fade","background_slideshow_transition_duration":500,"background_slideshow_background_size":"","background_slideshow_background_size_tablet":"","background_slideshow_background_size_mobile_extra":"","background_slideshow_background_size_mobile":"","background_slideshow_background_position":"","background_slideshow_background_position_tablet":"","background_slideshow_background_position_mobile_extra":"","background_slideshow_background_position_mobile":"","background_slideshow_lazyload":"","background_slideshow_ken_burns":"","background_slideshow_ken_burns_zoom_direction":"in","handle_slideshow_asset_loading":"","background_hover_background":"","background_hover_color":"","background_hover_color_stop":{"unit":"%","size":0,"sizes":[]},"background_hover_color_stop_tablet":{"unit":"%"},"background_hover_color_stop_mobile_extra":{"unit":"%"},"background_hover_color_stop_mobile":{"unit":"%"},"background_hover_color_b":"#f2295b","background_hover_color_b_stop":{"unit":"%","size":100,"sizes":[]},"background_hover_color_b_stop_tablet":{"unit":"%"},"background_hover_color_b_stop_mobile_extra":{"unit":"%"},"background_hover_color_b_stop_mobile":{"unit":"%"},"background_hover_gradient_type":"linear","background_hover_gradient_angle":{"unit":"deg","size":180,"sizes":[]},"background_hover_gradient_angle_tablet":{"unit":"deg"},"background_hover_gradient_angle_mobile_extra":{"unit":"deg"},"background_hover_gradient_angle_mobile":{"unit":"deg"},"background_hover_gradient_position":"center center","background_hover_gradient_position_tablet":"","background_hover_gradient_position_mobile_extra":"","background_hover_gradient_position_mobile":"","background_hover_image":{"url":"","id":"","size":""},"background_hover_image_tablet":{"url":"","id":"","size":""},"background_hover_image_mobile_extra":{"url":"","id":"","size":""},"background_hover_image_mobile":{"url":"","id":"","size":""},"background_hover_position":"","background_hover_position_tablet":"","background_hover_position_mobile_extra":"","background_hover_position_mobile":"","background_hover_xpos":{"unit":"px","size":0,"sizes":[]},"background_hover_xpos_tablet":{"unit":"px","size":0,"sizes":[]},"background_hover_xpos_mobile_extra":{"unit":"px","size":"","sizes":[]},"background_hover_xpos_mobile":{"unit":"px","size":0,"sizes":[]},"background_hover_ypos":{"unit":"px","size":0,"sizes":[]},"background_hover_ypos_tablet":{"unit":"px","size":0,"sizes":[]},"background_hover_ypos_mobile_extra":{"unit":"px","size":"","sizes":[]},"background_hover_ypos_mobile":{"unit":"px","size":0,"sizes":[]},"background_hover_attachment":"","background_hover_repeat":"","background_hover_repeat_tablet":"","background_hover_repeat_mobile_extra":"","background_hover_repeat_mobile":"","background_hover_size":"","background_hover_size_tablet":"","background_hover_size_mobile_extra":"","background_hover_size_mobile":"","background_hover_bg_width":{"unit":"%","size":100,"sizes":[]},"background_hover_bg_width_tablet":{"unit":"px","size":"","sizes":[]},"background_hover_bg_width_mobile_extra":{"unit":"px","size":"","sizes":[]},"background_hover_bg_width_mobile":{"unit":"px","size":"","sizes":[]},"background_hover_video_link":"","background_hover_video_start":"","background_hover_video_end":"","background_hover_play_once":"","background_hover_play_on_mobile":"","background_hover_privacy_mode":"","background_hover_video_fallback":{"url":"","id":"","size":""},"background_hover_slideshow_gallery":[],"background_hover_slideshow_loop":"yes","background_hover_slideshow_slide_duration":5000,"background_hover_slideshow_slide_transition":"fade","background_hover_slideshow_transition_duration":500,"background_hover_slideshow_background_size":"","background_hover_slideshow_background_size_tablet":"","background_hover_slideshow_background_size_mobile_extra":"","background_hover_slideshow_background_size_mobile":"","background_hover_slideshow_background_position":"","background_hover_slideshow_background_position_tablet":"","background_hover_slideshow_background_position_mobile_extra":"","background_hover_slideshow_background_position_mobile":"","background_hover_slideshow_lazyload":"","background_hover_slideshow_ken_burns":"","background_hover_slideshow_ken_burns_zoom_direction":"in","background_hover_transition":{"unit":"px","size":0.3,"sizes":[]},"background_overlay_background":"","background_overlay_color":"","background_overlay_color_stop":{"unit":"%","size":0,"sizes":[]},"background_overlay_color_stop_tablet":{"unit":"%"},"background_overlay_color_stop_mobile_extra":{"unit":"%"},"background_overlay_color_stop_mobile":{"unit":"%"},"background_overlay_color_b":"#f2295b","background_overlay_color_b_stop":{"unit":"%","size":100,"sizes":[]},"background_overlay_color_b_stop_tablet":{"unit":"%"},"background_overlay_color_b_stop_mobile_extra":{"unit":"%"},"background_overlay_color_b_stop_mobile":{"unit":"%"},"background_overlay_gradient_type":"linear","background_overlay_gradient_angle":{"unit":"deg","size":180,"sizes":[]},"background_overlay_gradient_angle_tablet":{"unit":"deg"},"background_overlay_gradient_angle_mobile_extra":{"unit":"deg"},"background_overlay_gradient_angle_mobile":{"unit":"deg"},"background_overlay_gradient_position":"center center","background_overlay_gradient_position_tablet":"","background_overlay_gradient_position_mobile_extra":"","background_overlay_gradient_position_mobile":"","background_overlay_image":{"url":"","id":"","size":""},"background_overlay_image_tablet":{"url":"","id":"","size":""},"background_overlay_image_mobile_extra":{"url":"","id":"","size":""},"background_overlay_image_mobile":{"url":"","id":"","size":""},"background_overlay_position":"","background_overlay_position_tablet":"","background_overlay_position_mobile_extra":"","background_overlay_position_mobile":"","background_overlay_xpos":{"unit":"px","size":0,"sizes":[]},"background_overlay_xpos_tablet":{"unit":"px","size":0,"sizes":[]},"background_overlay_xpos_mobile_extra":{"unit":"px","size":"","sizes":[]},"background_overlay_xpos_mobile":{"unit":"px","size":0,"sizes":[]},"background_overlay_ypos":{"unit":"px","size":0,"sizes":[]},"background_overlay_ypos_tablet":{"unit":"px","size":0,"sizes":[]},"background_overlay_ypos_mobile_extra":{"unit":"px","size":"","sizes":[]},"background_overlay_ypos_mobile":{"unit":"px","size":0,"sizes":[]},"background_overlay_attachment":"","background_overlay_repeat":"","background_overlay_repeat_tablet":"","background_overlay_repeat_mobile_extra":"","background_overlay_repeat_mobile":"","background_overlay_size":"","background_overlay_size_tablet":"","background_overlay_size_mobile_extra":"","background_overlay_size_mobile":"","background_overlay_bg_width":{"unit":"%","size":100,"sizes":[]},"background_overlay_bg_width_tablet":{"unit":"px","size":"","sizes":[]},"background_overlay_bg_width_mobile_extra":{"unit":"px","size":"","sizes":[]},"background_overlay_bg_width_mobile":{"unit":"px","size":"","sizes":[]},"background_overlay_video_link":"","background_overlay_video_start":"","background_overlay_video_end":"","background_overlay_play_once":"","background_overlay_play_on_mobile":"","background_overlay_privacy_mode":"","background_overlay_video_fallback":{"url":"","id":"","size":""},"background_overlay_slideshow_gallery":[],"background_overlay_slideshow_loop":"yes","background_overlay_slideshow_slide_duration":5000,"background_overlay_slideshow_slide_transition":"fade","background_overlay_slideshow_transition_duration":500,"background_overlay_slideshow_background_size":"","background_overlay_slideshow_background_size_tablet":"","background_overlay_slideshow_background_size_mobile_extra":"","background_overlay_slideshow_background_size_mobile":"","background_overlay_slideshow_background_position":"","background_overlay_slideshow_background_position_tablet":"","background_overlay_slideshow_background_position_mobile_extra":"","background_overlay_slideshow_background_position_mobile":"","background_overlay_slideshow_lazyload":"","background_overlay_slideshow_ken_burns":"","background_overlay_slideshow_ken_burns_zoom_direction":"in","background_overlay_opacity":{"unit":"px","size":0.5,"sizes":[]},"background_overlay_opacity_tablet":{"unit":"px","size":"","sizes":[]},"background_overlay_opacity_mobile_extra":{"unit":"px","size":"","sizes":[]},"background_overlay_opacity_mobile":{"unit":"px","size":"","sizes":[]},"css_filters_css_filter":"","css_filters_blur":{"unit":"px","size":0,"sizes":[]},"css_filters_brightness":{"unit":"px","size":100,"sizes":[]},"css_filters_contrast":{"unit":"px","size":100,"sizes":[]},"css_filters_saturate":{"unit":"px","size":100,"sizes":[]},"css_filters_hue":{"unit":"px","size":0,"sizes":[]},"overlay_blend_mode":"","background_overlay_hover_background":"","background_overlay_hover_color":"","background_overlay_hover_color_stop":{"unit":"%","size":0,"sizes":[]},"background_overlay_hover_color_stop_tablet":{"unit":"%"},"background_overlay_hover_color_stop_mobile_extra":{"unit":"%"},"background_overlay_hover_color_stop_mobile":{"unit":"%"},"background_overlay_hover_color_b":"#f2295b","background_overlay_hover_color_b_stop":{"unit":"%","size":100,"sizes":[]},"background_overlay_hover_color_b_stop_tablet":{"unit":"%"},"background_overlay_hover_color_b_stop_mobile_extra":{"unit":"%"},"background_overlay_hover_color_b_stop_mobile":{"unit":"%"},"background_overlay_hover_gradient_type":"linear","background_overlay_hover_gradient_angle":{"unit":"deg","size":180,"sizes":[]},"background_overlay_hover_gradient_angle_tablet":{"unit":"deg"},"background_overlay_hover_gradient_angle_mobile_extra":{"unit":"deg"},"background_overlay_hover_gradient_angle_mobile":{"unit":"deg"},"background_overlay_hover_gradient_position":"center center","background_overlay_hover_gradient_position_tablet":"","background_overlay_hover_gradient_position_mobile_extra":"","background_overlay_hover_gradient_position_mobile":"","background_overlay_hover_image":{"url":"","id":"","size":""},"background_overlay_hover_image_tablet":{"url":"","id":"","size":""},"background_overlay_hover_image_mobile_extra":{"url":"","id":"","size":""},"background_overlay_hover_image_mobile":{"url":"","id":"","size":""},"background_overlay_hover_position":"","background_overlay_hover_position_tablet":"","background_overlay_hover_position_mobile_extra":"","background_overlay_hover_position_mobile":"","background_overlay_hover_xpos":{"unit":"px","size":0,"sizes":[]},"background_overlay_hover_xpos_tablet":{"unit":"px","size":0,"sizes":[]},"background_overlay_hover_xpos_mobile_extra":{"unit":"px","size":"","sizes":[]},"background_overlay_hover_xpos_mobile":{"unit":"px","size":0,"sizes":[]},"background_overlay_hover_ypos":{"unit":"px","size":0,"sizes":[]},"background_overlay_hover_ypos_tablet":{"unit":"px","size":0,"sizes":[]},"background_overlay_hover_ypos_mobile_extra":{"unit":"px","size":"","sizes":[]},"background_overlay_hover_ypos_mobile":{"unit":"px","size":0,"sizes":[]},"background_overlay_hover_attachment":"","background_overlay_hover_repeat":"","background_overlay_hover_repeat_tablet":"","background_overlay_hover_repeat_mobile_extra":"","background_overlay_hover_repeat_mobile":"","background_overlay_hover_size":"","background_overlay_hover_size_tablet":"","background_overlay_hover_size_mobile_extra":"","background_overlay_hover_size_mobile":"","background_overlay_hover_bg_width":{"unit":"%","size":100,"sizes":[]},"background_overlay_hover_bg_width_tablet":{"unit":"px","size":"","sizes":[]},"background_overlay_hover_bg_width_mobile_extra":{"unit":"px","size":"","sizes":[]},"background_overlay_hover_bg_width_mobile":{"unit":"px","size":"","sizes":[]},"background_overlay_hover_video_link":"","background_overlay_hover_video_start":"","background_overlay_hover_video_end":"","background_overlay_hover_play_once":"","background_overlay_hover_play_on_mobile":"","background_overlay_hover_privacy_mode":"","background_overlay_hover_video_fallback":{"url":"","id":"","size":""},"background_overlay_hover_slideshow_gallery":[],"background_overlay_hover_slideshow_loop":"yes","background_overlay_hover_slideshow_slide_duration":5000,"background_overlay_hover_slideshow_slide_transition":"fade","background_overlay_hover_slideshow_transition_duration":500,"background_overlay_hover_slideshow_background_size":"","background_overlay_hover_slideshow_background_size_tablet":"","background_overlay_hover_slideshow_background_size_mobile_extra":"","background_overlay_hover_slideshow_background_size_mobile":"","background_overlay_hover_slideshow_background_position":"","background_overlay_hover_slideshow_background_position_tablet":"","background_overlay_hover_slideshow_background_position_mobile_extra":"","background_overlay_hover_slideshow_background_position_mobile":"","background_overlay_hover_slideshow_lazyload":"","background_overlay_hover_slideshow_ken_burns":"","background_overlay_hover_slideshow_ken_burns_zoom_direction":"in","background_overlay_hover_opacity":{"unit":"px","size":0.5,"sizes":[]},"background_overlay_hover_opacity_tablet":{"unit":"px","size":"","sizes":[]},"background_overlay_hover_opacity_mobile_extra":{"unit":"px","size":"","sizes":[]},"background_overlay_hover_opacity_mobile":{"unit":"px","size":"","sizes":[]},"background_overlay_hover_transition":{"unit":"px","size":"","sizes":[]},"css_filters_hover_css_filter":"","css_filters_hover_blur":{"unit":"px","size":0,"sizes":[]},"css_filters_hover_brightness":{"unit":"px","size":100,"sizes":[]},"css_filters_hover_contrast":{"unit":"px","size":100,"sizes":[]},"css_filters_hover_saturate":{"unit":"px","size":100,"sizes":[]},"css_filters_hover_hue":{"unit":"px","size":0,"sizes":[]},"border_border":"","border_width":{"unit":"px","top":"","right":"","bottom":"","left":"","isLinked":true},"border_width_tablet":{"unit":"px","top":"","right":"","bottom":"","left":"","isLinked":true},"border_width_mobile_extra":{"unit":"px","top":"","right":"","bottom":"","left":"","isLinked":true},"border_width_mobile":{"unit":"px","top":"","right":"","bottom":"","left":"","isLinked":true},"border_color":"","border_radius":{"unit":"px","top":"","right":"","bottom":"","left":"","isLinked":true},"border_radius_tablet":{"unit":"px","top":"","right":"","bottom":"","left":"","isLinked":true},"border_radius_mobile_extra":{"unit":"px","top":"","right":"","bottom":"","left":"","isLinked":true},"border_radius_mobile":{"unit":"px","top":"","right":"","bottom":"","left":"","isLinked":true},"box_shadow_box_shadow_type":"","box_shadow_box_shadow":{"horizontal":0,"vertical":0,"blur":10,"spread":0,"color":"rgba(0,0,0,0.5)"},"box_shadow_box_shadow_position":" ","border_hover_border":"","border_hover_width":{"unit":"px","top":"","right":"","bottom":"","left":"","isLinked":true},"border_hover_width_tablet":{"unit":"px","top":"","right":"","bottom":"","left":"","isLinked":true},"border_hover_width_mobile_extra":{"unit":"px","top":"","right":"","bottom":"","left":"","isLinked":true},"border_hover_width_mobile":{"unit":"px","top":"","right":"","bottom":"","left":"","isLinked":true},"border_hover_color":"","border_radius_hover":{"unit":"px","top":"","right":"","bottom":"","left":"","isLinked":true},"border_radius_hover_tablet":{"unit":"px","top":"","right":"","bottom":"","left":"","isLinked":true},"border_radius_hover_mobile_extra":{"unit":"px","top":"","right":"","bottom":"","left":"","isLinked":true},"border_radius_hover_mobile":{"unit":"px","top":"","right":"","bottom":"","left":"","isLinked":true},"box_shadow_hover_box_shadow_type":"","box_shadow_hover_box_shadow":{"horizontal":0,"vertical":0,"blur":10,"spread":0,"color":"rgba(0,0,0,0.5)"},"box_shadow_hover_box_shadow_position":" ","border_hover_transition":{"unit":"px","size":0.3,"sizes":[]},"shape_divider_top":"","shape_divider_top_color":"","shape_divider_top_width":{"unit":"%","size":"","sizes":[]},"shape_divider_top_width_tablet":{"unit":"%","size":"","sizes":[]},"shape_divider_top_width_mobile_extra":{"unit":"px","size":"","sizes":[]},"shape_divider_top_width_mobile":{"unit":"%","size":"","sizes":[]},"shape_divider_top_height":{"unit":"px","size":"","sizes":[]},"shape_divider_top_height_tablet":{"unit":"px","size":"","sizes":[]},"shape_divider_top_height_mobile_extra":{"unit":"px","size":"","sizes":[]},"shape_divider_top_height_mobile":{"unit":"px","size":"","sizes":[]},"shape_divider_top_flip":"","shape_divider_top_negative":"","shape_divider_top_above_content":"","shape_divider_bottom":"","shape_divider_bottom_color":"","shape_divider_bottom_width":{"unit":"%","size":"","sizes":[]},"shape_divider_bottom_width_tablet":{"unit":"%","size":"","sizes":[]},"shape_divider_bottom_width_mobile_extra":{"unit":"px","size":"","sizes":[]},"shape_divider_bottom_width_mobile":{"unit":"%","size":"","sizes":[]},"shape_divider_bottom_height":{"unit":"px","size":"","sizes":[]},"shape_divider_bottom_height_tablet":{"unit":"px","size":"","sizes":[]},"shape_divider_bottom_height_mobile_extra":{"unit":"px","size":"","sizes":[]},"shape_divider_bottom_height_mobile":{"unit":"px","size":"","sizes":[]},"shape_divider_bottom_flip":"","shape_divider_bottom_negative":"","shape_divider_bottom_above_content":"","margin":{"unit":"px","top":"","right":"","bottom":"","left":"","isLinked":true},"margin_tablet":{"unit":"px","top":"","right":"","bottom":"","left":"","isLinked":true},"margin_mobile_extra":{"unit":"px","top":"","right":"","bottom":"","left":"","isLinked":true},"margin_mobile":{"unit":"px","top":"","right":"","bottom":"","left":"","isLinked":true},"padding":{"unit":"px","top":"","right":"","bottom":"","left":"","isLinked":true},"padding_tablet":{"unit":"px","top":"","right":"","bottom":"","left":"","isLinked":true},"padding_mobile_extra":{"unit":"px","top":"","right":"","bottom":"","left":"","isLinked":true},"padding_mobile":{"unit":"px","top":"","right":"","bottom":"","left":"","isLinked":true},"grid_column":"","grid_column_tablet":"","grid_column_mobile_extra":"","grid_column_mobile":"","grid_column_custom":"","grid_column_custom_tablet":"","grid_column_custom_mobile_extra":"","grid_column_custom_mobile":"","grid_row":"","grid_row_tablet":"","grid_row_mobile_extra":"","grid_row_mobile":"","grid_row_custom":"","grid_row_custom_tablet":"","grid_row_custom_mobile_extra":"","grid_row_custom_mobile":"","_flex_align_self":"","_flex_align_self_tablet":"","_flex_align_self_mobile_extra":"","_flex_align_self_mobile":"","_flex_order":"","_flex_order_tablet":"","_flex_order_mobile_extra":"","_flex_order_mobile":"","_flex_order_custom":"","_flex_order_custom_tablet":"","_flex_order_custom_mobile_extra":"","_flex_order_custom_mobile":"","_flex_size":"","_flex_size_tablet":"","_flex_size_mobile_extra":"","_flex_size_mobile":"","_flex_grow":1,"_flex_grow_tablet":"","_flex_grow_mobile_extra":"","_flex_grow_mobile":"","_flex_shrink":1,"_flex_shrink_tablet":"","_flex_shrink_mobile_extra":"","_flex_shrink_mobile":"","position":"","_offset_orientation_h":"start","_offset_x":{"unit":"px","size":0,"sizes":[]},"_offset_x_tablet":{"unit":"px","size":"","sizes":[]},"_offset_x_mobile_extra":{"unit":"px","size":"","sizes":[]},"_offset_x_mobile":{"unit":"px","size":"","sizes":[]},"_offset_x_end":{"unit":"px","size":0,"sizes":[]},"_offset_x_end_tablet":{"unit":"px","size":"","sizes":[]},"_offset_x_end_mobile_extra":{"unit":"px","size":"","sizes":[]},"_offset_x_end_mobile":{"unit":"px","size":"","sizes":[]},"_offset_orientation_v":"start","_offset_y":{"unit":"px","size":0,"sizes":[]},"_offset_y_tablet":{"unit":"px","size":"","sizes":[]},"_offset_y_mobile_extra":{"unit":"px","size":"","sizes":[]},"_offset_y_mobile":{"unit":"px","size":"","sizes":[]},"_offset_y_end":{"unit":"px","size":0,"sizes":[]},"_offset_y_end_tablet":{"unit":"px","size":"","sizes":[]},"_offset_y_end_mobile_extra":{"unit":"px","size":"","sizes":[]},"_offset_y_end_mobile":{"unit":"px","size":"","sizes":[]},"z_index":"","z_index_tablet":"","z_index_mobile_extra":"","z_index_mobile":"","_element_id":"","css_classes":"","animation_type":"","transform_animated_type":"","infinite_animation_type":"","enable_custom_duration":"","custom_duration":{"unit":"px","size":"","sizes":[]},"custom_duration_tablet":{"unit":"px","size":"","sizes":[]},"custom_duration_mobile_extra":{"unit":"px","size":"","sizes":[]},"custom_duration_mobile":{"unit":"px","size":"","sizes":[]},"custom_count":{"unit":"px","size":"","sizes":[]},"custom_count_tablet":{"unit":"px","size":"","sizes":[]},"custom_count_mobile_extra":{"unit":"px","size":"","sizes":[]},"custom_count_mobile":{"unit":"px","size":"","sizes":[]},"item_opacity":{"unit":"px","size":"","sizes":[]},"item_opacity_tablet":{"unit":"px","size":"","sizes":[]},"item_opacity_mobile_extra":{"unit":"px","size":"","sizes":[]},"item_opacity_mobile":{"unit":"px","size":"","sizes":[]},"wpr_enable_particles":"no","which_particle":"wpr_particle_json_custom","wpr_particle_json_custom":"{\"particles\":{\"number\":{\"value\":80,\"density\":{\"enable\":true,\"value_area\":800}},\"color\":{\"value\":\"#000000\"},\"shape\":{\"type\":\"circle\",\"stroke\":{\"width\":0,\"color\":\"#000000\"},\"polygon\":{\"nb_sides\":5},\"image\":{\"src\":\"img/github.svg\",\"width\":100,\"height\":100}},\"opacity\":{\"value\":0.5,\"random\":false,\"anim\":{\"enable\":false,\"speed\":1,\"opacity_min\":0.1,\"sync\":false}},\"size\":{\"value\":3,\"random\":true,\"anim\":{\"enable\":false,\"speed\":40,\"size_min\":0.1,\"sync\":false}},\"line_linked\":{\"enable\":true,\"distance\":150,\"color\":\"#000000\",\"opacity\":0.4,\"width\":1},\"move\":{\"enable\":true,\"speed\":6,\"direction\":\"none\",\"random\":false,\"straight\":false,\"out_mode\":\"out\",\"bounce\":false,\"attract\":{\"enable\":false,\"rotateX\":600,\"rotateY\":1200}}},\"interactivity\":{\"detect_on\":\"window\",\"events\":{\"onhover\":{\"enable\":true,\"mode\":\"repulse\"},\"onclick\":{\"enable\":true,\"mode\":\"push\"},\"resize\":true},\"modes\":{\"grab\":{\"distance\":400,\"line_linked\":{\"opacity\":1}},\"bubble\":{\"distance\":400,\"size\":40,\"duration\":2,\"opacity\":8,\"speed\":3},\"repulse\":{\"distance\":200,\"duration\":0.4},\"push\":{\"particles_nb\":4},\"remove\":{\"particles_nb\":2}}},\"retina_detect\":true}","wpr_enable_jarallax":"no","speed":1.4,"scroll_effect":"scroll","bg_image":{"url":"https://thisislocomotive.com/wp-content/plugins/elementor/assets/images/placeholder.png","id":"","size":""},"wpr_enable_parallax_hover":"no","invert_direction":"no","scalar_speed":{"unit":"%","size":10,"sizes":[]},"enable_sticky_section":"no","enable_on_devices":["desktop_sticky"],"position_type":"sticky","sticky_type":"sticky","position_location":"top","position_offset":0,"position_offset_tablet":0,"position_offset_mobile_extra":0,"position_offset_mobile":0,"wpr_z_index":10,"custom_breakpoints":false,"active_breakpoints":["mobile_sticky","mobile_extra_sticky","tablet_sticky","desktop_sticky"],"animation":"","animation_tablet":"","animation_mobile_extra":"","animation_mobile":"","animation_duration":"","animation_delay":"","thim_ekit_motion_fx_mouse":"","thim_ekit_motion_fx_mouseTrack_effect":"","thim_ekit_motion_fx_mouseTrack_direction":"","thim_ekit_motion_fx_mouseTrack_speed":{"unit":"px","size":1,"sizes":[]},"thim_ekit_motion_fx_tilt_effect":"","thim_ekit_motion_fx_tilt_direction":"","thim_ekit_motion_fx_tilt_speed":{"unit":"px","size":1,"sizes":[]},"_transform_rotate_popover":"","_transform_rotateZ_effect":{"unit":"px","size":"","sizes":[]},"_transform_rotateZ_effect_tablet":{"unit":"deg","size":"","sizes":[]},"_transform_rotateZ_effect_mobile_extra":{"unit":"deg","size":"","sizes":[]},"_transform_rotateZ_effect_mobile":{"unit":"deg","size":"","sizes":[]},"_transform_rotate_3d":"","_transform_rotateX_effect":{"unit":"px","size":"","sizes":[]},"_transform_rotateX_effect_tablet":{"unit":"deg","size":"","sizes":[]},"_transform_rotateX_effect_mobile_extra":{"unit":"deg","size":"","sizes":[]},"_transform_rotateX_effect_mobile":{"unit":"deg","size":"","sizes":[]},"_transform_rotateY_effect":{"unit":"px","size":"","sizes":[]},"_transform_rotateY_effect_tablet":{"unit":"deg","size":"","sizes":[]},"_transform_rotateY_effect_mobile_extra":{"unit":"deg","size":"","sizes":[]},"_transform_rotateY_effect_mobile":{"unit":"deg","size":"","sizes":[]},"_transform_perspective_effect":{"unit":"px","size":"","sizes":[]},"_transform_perspective_effect_tablet":{"unit":"px","size":"","sizes":[]},"_transform_perspective_effect_mobile_extra":{"unit":"px","size":"","sizes":[]},"_transform_perspective_effect_mobile":{"unit":"px","size":"","sizes":[]},"_transform_translate_popover":"","_transform_translateX_effect":{"unit":"px","size":"","sizes":[]},"_transform_translateX_effect_tablet":{"unit":"px","size":"","sizes":[]},"_transform_translateX_effect_mobile_extra":{"unit":"px","size":"","sizes":[]},"_transform_translateX_effect_mobile":{"unit":"px","size":"","sizes":[]},"_transform_translateY_effect":{"unit":"px","size":"","sizes":[]},"_transform_translateY_effect_tablet":{"unit":"px","size":"","sizes":[]},"_transform_translateY_effect_mobile_extra":{"unit":"px","size":"","sizes":[]},"_transform_translateY_effect_mobile":{"unit":"px","size":"","sizes":[]},"_transform_scale_popover":"","_transform_keep_proportions":"yes","_transform_scale_effect":{"unit":"px","size":"","sizes":[]},"_transform_scale_effect_tablet":{"unit":"px","size":"","sizes":[]},"_transform_scale_effect_mobile_extra":{"unit":"px","size":"","sizes":[]},"_transform_scale_effect_mobile":{"unit":"px","size":"","sizes":[]},"_transform_scaleX_effect":{"unit":"px","size":"","sizes":[]},"_transform_scaleX_effect_tablet":{"unit":"px","size":"","sizes":[]},"_transform_scaleX_effect_mobile_extra":{"unit":"px","size":"","sizes":[]},"_transform_scaleX_effect_mobile":{"unit":"px","size":"","sizes":[]},"_transform_scaleY_effect":{"unit":"px","size":"","sizes":[]},"_transform_scaleY_effect_tablet":{"unit":"px","size":"","sizes":[]},"_transform_scaleY_effect_mobile_extra":{"unit":"px","size":"","sizes":[]},"_transform_scaleY_effect_mobile":{"unit":"px","size":"","sizes":[]},"_transform_skew_popover":"","_transform_skewX_effect":{"unit":"px","size":"","sizes":[]},"_transform_skewX_effect_tablet":{"unit":"deg","size":"","sizes":[]},"_transform_skewX_effect_mobile_extra":{"unit":"deg","size":"","sizes":[]},"_transform_skewX_effect_mobile":{"unit":"deg","size":"","sizes":[]},"_transform_skewY_effect":{"unit":"px","size":"","sizes":[]},"_transform_skewY_effect_tablet":{"unit":"deg","size":"","sizes":[]},"_transform_skewY_effect_mobile_extra":{"unit":"deg","size":"","sizes":[]},"_transform_skewY_effect_mobile":{"unit":"deg","size":"","sizes":[]},"_transform_flipX_effect":"","_transform_flipY_effect":"","_transform_rotate_popover_hover":"","_transform_rotateZ_effect_hover":{"unit":"px","size":"","sizes":[]},"_transform_rotateZ_effect_hover_tablet":{"unit":"deg","size":"","sizes":[]},"_transform_rotateZ_effect_hover_mobile_extra":{"unit":"deg","size":"","sizes":[]},"_transform_rotateZ_effect_hover_mobile":{"unit":"deg","size":"","sizes":[]},"_transform_rotate_3d_hover":"","_transform_rotateX_effect_hover":{"unit":"px","size":"","sizes":[]},"_transform_rotateX_effect_hover_tablet":{"unit":"deg","size":"","sizes":[]},"_transform_rotateX_effect_hover_mobile_extra":{"unit":"deg","size":"","sizes":[]},"_transform_rotateX_effect_hover_mobile":{"unit":"deg","size":"","sizes":[]},"_transform_rotateY_effect_hover":{"unit":"px","size":"","sizes":[]},"_transform_rotateY_effect_hover_tablet":{"unit":"deg","size":"","sizes":[]},"_transform_rotateY_effect_hover_mobile_extra":{"unit":"deg","size":"","sizes":[]},"_transform_rotateY_effect_hover_mobile":{"unit":"deg","size":"","sizes":[]},"_transform_perspective_effect_hover":{"unit":"px","size":"","sizes":[]},"_transform_perspective_effect_hover_tablet":{"unit":"px","size":"","sizes":[]},"_transform_perspective_effect_hover_mobile_extra":{"unit":"px","size":"","sizes":[]},"_transform_perspective_effect_hover_mobile":{"unit":"px","size":"","sizes":[]},"_transform_translate_popover_hover":"","_transform_translateX_effect_hover":{"unit":"px","size":"","sizes":[]},"_transform_translateX_effect_hover_tablet":{"unit":"px","size":"","sizes":[]},"_transform_translateX_effect_hover_mobile_extra":{"unit":"px","size":"","sizes":[]},"_transform_translateX_effect_hover_mobile":{"unit":"px","size":"","sizes":[]},"_transform_translateY_effect_hover":{"unit":"px","size":"","sizes":[]},"_transform_translateY_effect_hover_tablet":{"unit":"px","size":"","sizes":[]},"_transform_translateY_effect_hover_mobile_extra":{"unit":"px","size":"","sizes":[]},"_transform_translateY_effect_hover_mobile":{"unit":"px","size":"","sizes":[]},"_transform_scale_popover_hover":"","_transform_keep_proportions_hover":"yes","_transform_scale_effect_hover":{"unit":"px","size":"","sizes":[]},"_transform_scale_effect_hover_tablet":{"unit":"px","size":"","sizes":[]},"_transform_scale_effect_hover_mobile_extra":{"unit":"px","size":"","sizes":[]},"_transform_scale_effect_hover_mobile":{"unit":"px","size":"","sizes":[]},"_transform_scaleX_effect_hover":{"unit":"px","size":"","sizes":[]},"_transform_scaleX_effect_hover_tablet":{"unit":"px","size":"","sizes":[]},"_transform_scaleX_effect_hover_mobile_extra":{"unit":"px","size":"","sizes":[]},"_transform_scaleX_effect_hover_mobile":{"unit":"px","size":"","sizes":[]},"_transform_scaleY_effect_hover":{"unit":"px","size":"","sizes":[]},"_transform_scaleY_effect_hover_tablet":{"unit":"px","size":"","sizes":[]},"_transform_scaleY_effect_hover_mobile_extra":{"unit":"px","size":"","sizes":[]},"_transform_scaleY_effect_hover_mobile":{"unit":"px","size":"","sizes":[]},"_transform_skew_popover_hover":"","_transform_skewX_effect_hover":{"unit":"px","size":"","sizes":[]},"_transform_skewX_effect_hover_tablet":{"unit":"deg","size":"","sizes":[]},"_transform_skewX_effect_hover_mobile_extra":{"unit":"deg","size":"","sizes":[]},"_transform_skewX_effect_hover_mobile":{"unit":"deg","size":"","sizes":[]},"_transform_skewY_effect_hover":{"unit":"px","size":"","sizes":[]},"_transform_skewY_effect_hover_tablet":{"unit":"deg","size":"","sizes":[]},"_transform_skewY_effect_hover_mobile_extra":{"unit":"deg","size":"","sizes":[]},"_transform_skewY_effect_hover_mobile":{"unit":"deg","size":"","sizes":[]},"_transform_flipX_effect_hover":"","_transform_flipY_effect_hover":"","_transform_transition_hover":{"unit":"px","size":"","sizes":[]},"motion_fx_transform_x_anchor_point":"","motion_fx_transform_x_anchor_point_tablet":"","motion_fx_transform_x_anchor_point_mobile_extra":"","motion_fx_transform_x_anchor_point_mobile":"","motion_fx_transform_y_anchor_point":"","motion_fx_transform_y_anchor_point_tablet":"","motion_fx_transform_y_anchor_point_mobile_extra":"","motion_fx_transform_y_anchor_point_mobile":"","hide_desktop":"","hide_tablet":"","hide_mobile_extra":"","hide_mobile":"","custom_css":"","wpr_custom_css":""},"defaultEditSettings":{"defaultEditRoute":"content"},"elements":[{"id":"197a027","elType":"widget","isInner":false,"isLocked":false,"settings":{"title":"
IMAT 2024 File (only Pictures) – Download Now
","header_size":"h6","align":"justify","typography_typography":"custom","typography_font_family":"Barlow","typography_font_size":{"unit":"px","size":16,"sizes":[]},"typography_font_weight":"600","typography_text_transform":"none","typography_line_height":{"unit":"em","size":1.4,"sizes":[]},"typography_letter_spacing":{"unit":"px","size":0,"sizes":[]},"typography_word_spacing":{"unit":"px","size":0,"sizes":[]},"text_stroke_text_stroke":{"unit":"px","size":0,"sizes":[]},"title_color":"#000000","__globals__":{"typography_typography":""},"link":{"url":"https://thisislocomotive.com/wp-content/uploads/2025/07/Image_2024_merged.pdf","is_external":"","nofollow":"","custom_attributes":""},"size":"default","animated_headline_promotion":"","align_tablet":"","align_mobile_extra":"","align_mobile":"","typography_font_size_tablet":{"unit":"px","size":"","sizes":[]},"typography_font_size_mobile_extra":{"unit":"px","size":"","sizes":[]},"typography_font_size_mobile":{"unit":"px","size":"","sizes":[]},"typography_font_style":"","typography_text_decoration":"","typography_line_height_tablet":{"unit":"em","size":"","sizes":[]},"typography_line_height_mobile_extra":{"unit":"px","size":"","sizes":[]},"typography_line_height_mobile":{"unit":"em","size":"","sizes":[]},"typography_letter_spacing_tablet":{"unit":"px","size":"","sizes":[]},"typography_letter_spacing_mobile_extra":{"unit":"px","size":"","sizes":[]},"typography_letter_spacing_mobile":{"unit":"px","size":"","sizes":[]},"typography_word_spacing_tablet":{"unit":"em","size":"","sizes":[]},"typography_word_spacing_mobile_extra":{"unit":"px","size":"","sizes":[]},"typography_word_spacing_mobile":{"unit":"em","size":"","sizes":[]},"text_stroke_text_stroke_type":"","text_stroke_text_stroke_tablet":{"unit":"px","size":"","sizes":[]},"text_stroke_text_stroke_mobile_extra":{"unit":"px","size":"","sizes":[]},"text_stroke_text_stroke_mobile":{"unit":"px","size":"","sizes":[]},"text_stroke_stroke_color":"#000","text_shadow_text_shadow_type":"","text_shadow_text_shadow":{"horizontal":0,"vertical":0,"blur":10,"color":"rgba(0,0,0,0.3)"},"blend_mode":"","title_hover_color":"","title_hover_color_transition_duration":{"unit":"s","size":"","sizes":[]},"_title":"","_margin":{"unit":"px","top":"","right":"","bottom":"","left":"","isLinked":true},"_margin_tablet":{"unit":"px","top":"","right":"","bottom":"","left":"","isLinked":true},"_margin_mobile_extra":{"unit":"px","top":"","right":"","bottom":"","left":"","isLinked":true},"_margin_mobile":{"unit":"px","top":"","right":"","bottom":"","left":"","isLinked":true},"_padding":{"unit":"px","top":"","right":"","bottom":"","left":"","isLinked":true},"_padding_tablet":{"unit":"px","top":"","right":"","bottom":"","left":"","isLinked":true},"_padding_mobile_extra":{"unit":"px","top":"","right":"","bottom":"","left":"","isLinked":true},"_padding_mobile":{"unit":"px","top":"","right":"","bottom":"","left":"","isLinked":true},"_element_width":"","_element_width_tablet":"","_element_width_mobile_extra":"","_element_width_mobile":"","_element_custom_width":{"unit":"%","size":"","sizes":[]},"_element_custom_width_tablet":{"unit":"px","size":"","sizes":[]},"_element_custom_width_mobile_extra":{"unit":"px","size":"","sizes":[]},"_element_custom_width_mobile":{"unit":"px","size":"","sizes":[]},"_grid_column":"","_grid_column_tablet":"","_grid_column_mobile_extra":"","_grid_column_mobile":"","_grid_column_custom":"","_grid_column_custom_tablet":"","_grid_column_custom_mobile_extra":"","_grid_column_custom_mobile":"","_grid_row":"","_grid_row_tablet":"","_grid_row_mobile_extra":"","_grid_row_mobile":"","_grid_row_custom":"","_grid_row_custom_tablet":"","_grid_row_custom_mobile_extra":"","_grid_row_custom_mobile":"","_flex_align_self":"","_flex_align_self_tablet":"","_flex_align_self_mobile_extra":"","_flex_align_self_mobile":"","_flex_order":"","_flex_order_tablet":"","_flex_order_mobile_extra":"","_flex_order_mobile":"","_flex_order_custom":"","_flex_order_custom_tablet":"","_flex_order_custom_mobile_extra":"","_flex_order_custom_mobile":"","_flex_size":"","_flex_size_tablet":"","_flex_size_mobile_extra":"","_flex_size_mobile":"","_flex_grow":1,"_flex_grow_tablet":"","_flex_grow_mobile_extra":"","_flex_grow_mobile":"","_flex_shrink":1,"_flex_shrink_tablet":"","_flex_shrink_mobile_extra":"","_flex_shrink_mobile":"","_element_vertical_align":"","_element_vertical_align_tablet":"","_element_vertical_align_mobile_extra":"","_element_vertical_align_mobile":"","_position":"","_offset_orientation_h":"start","_offset_x":{"unit":"px","size":0,"sizes":[]},"_offset_x_tablet":{"unit":"px","size":"","sizes":[]},"_offset_x_mobile_extra":{"unit":"px","size":"","sizes":[]},"_offset_x_mobile":{"unit":"px","size":"","sizes":[]},"_offset_x_end":{"unit":"px","size":0,"sizes":[]},"_offset_x_end_tablet":{"unit":"px","size":"","sizes":[]},"_offset_x_end_mobile_extra":{"unit":"px","size":"","sizes":[]},"_offset_x_end_mobile":{"unit":"px","size":"","sizes":[]},"_offset_orientation_v":"start","_offset_y":{"unit":"px","size":0,"sizes":[]},"_offset_y_tablet":{"unit":"px","size":"","sizes":[]},"_offset_y_mobile_extra":{"unit":"px","size":"","sizes":[]},"_offset_y_mobile":{"unit":"px","size":"","sizes":[]},"_offset_y_end":{"unit":"px","size":0,"sizes":[]},"_offset_y_end_tablet":{"unit":"px","size":"","sizes":[]},"_offset_y_end_mobile_extra":{"unit":"px","size":"","sizes":[]},"_offset_y_end_mobile":{"unit":"px","size":"","sizes":[]},"_z_index":"","_z_index_tablet":"","_z_index_mobile_extra":"","_z_index_mobile":"","_element_id":"","_css_classes":"","_animation":"","_animation_tablet":"","_animation_mobile_extra":"","_animation_mobile":"","animation_duration":"","_animation_delay":"","thim_ekit_motion_fx_mouse":"","thim_ekit_motion_fx_mouseTrack_effect":"","thim_ekit_motion_fx_mouseTrack_direction":"","thim_ekit_motion_fx_mouseTrack_speed":{"unit":"px","size":1,"sizes":[]},"thim_ekit_motion_fx_tilt_effect":"","thim_ekit_motion_fx_tilt_direction":"","thim_ekit_motion_fx_tilt_speed":{"unit":"px","size":1,"sizes":[]},"_transform_rotate_popover":"","_transform_rotateZ_effect":{"unit":"px","size":"","sizes":[]},"_transform_rotateZ_effect_tablet":{"unit":"deg","size":"","sizes":[]},"_transform_rotateZ_effect_mobile_extra":{"unit":"deg","size":"","sizes":[]},"_transform_rotateZ_effect_mobile":{"unit":"deg","size":"","sizes":[]},"_transform_rotate_3d":"","_transform_rotateX_effect":{"unit":"px","size":"","sizes":[]},"_transform_rotateX_effect_tablet":{"unit":"deg","size":"","sizes":[]},"_transform_rotateX_effect_mobile_extra":{"unit":"deg","size":"","sizes":[]},"_transform_rotateX_effect_mobile":{"unit":"deg","size":"","sizes":[]},"_transform_rotateY_effect":{"unit":"px","size":"","sizes":[]},"_transform_rotateY_effect_tablet":{"unit":"deg","size":"","sizes":[]},"_transform_rotateY_effect_mobile_extra":{"unit":"deg","size":"","sizes":[]},"_transform_rotateY_effect_mobile":{"unit":"deg","size":"","sizes":[]},"_transform_perspective_effect":{"unit":"px","size":"","sizes":[]},"_transform_perspective_effect_tablet":{"unit":"px","size":"","sizes":[]},"_transform_perspective_effect_mobile_extra":{"unit":"px","size":"","sizes":[]},"_transform_perspective_effect_mobile":{"unit":"px","size":"","sizes":[]},"_transform_translate_popover":"","_transform_translateX_effect":{"unit":"px","size":"","sizes":[]},"_transform_translateX_effect_tablet":{"unit":"px","size":"","sizes":[]},"_transform_translateX_effect_mobile_extra":{"unit":"px","size":"","sizes":[]},"_transform_translateX_effect_mobile":{"unit":"px","size":"","sizes":[]},"_transform_translateY_effect":{"unit":"px","size":"","sizes":[]},"_transform_translateY_effect_tablet":{"unit":"px","size":"","sizes":[]},"_transform_translateY_effect_mobile_extra":{"unit":"px","size":"","sizes":[]},"_transform_translateY_effect_mobile":{"unit":"px","size":"","sizes":[]},"_transform_scale_popover":"","_transform_keep_proportions":"yes","_transform_scale_effect":{"unit":"px","size":"","sizes":[]},"_transform_scale_effect_tablet":{"unit":"px","size":"","sizes":[]},"_transform_scale_effect_mobile_extra":{"unit":"px","size":"","sizes":[]},"_transform_scale_effect_mobile":{"unit":"px","size":"","sizes":[]},"_transform_scaleX_effect":{"unit":"px","size":"","sizes":[]},"_transform_scaleX_effect_tablet":{"unit":"px","size":"","sizes":[]},"_transform_scaleX_effect_mobile_extra":{"unit":"px","size":"","sizes":[]},"_transform_scaleX_effect_mobile":{"unit":"px","size":"","sizes":[]},"_transform_scaleY_effect":{"unit":"px","size":"","sizes":[]},"_transform_scaleY_effect_tablet":{"unit":"px","size":"","sizes":[]},"_transform_scaleY_effect_mobile_extra":{"unit":"px","size":"","sizes":[]},"_transform_scaleY_effect_mobile":{"unit":"px","size":"","sizes":[]},"_transform_skew_popover":"","_transform_skewX_effect":{"unit":"px","size":"","sizes":[]},"_transform_skewX_effect_tablet":{"unit":"deg","size":"","sizes":[]},"_transform_skewX_effect_mobile_extra":{"unit":"deg","size":"","sizes":[]},"_transform_skewX_effect_mobile":{"unit":"deg","size":"","sizes":[]},"_transform_skewY_effect":{"unit":"px","size":"","sizes":[]},"_transform_skewY_effect_tablet":{"unit":"deg","size":"","sizes":[]},"_transform_skewY_effect_mobile_extra":{"unit":"deg","size":"","sizes":[]},"_transform_skewY_effect_mobile":{"unit":"deg","size":"","sizes":[]},"_transform_flipX_effect":"","_transform_flipY_effect":"","_transform_rotate_popover_hover":"","_transform_rotateZ_effect_hover":{"unit":"px","size":"","sizes":[]},"_transform_rotateZ_effect_hover_tablet":{"unit":"deg","size":"","sizes":[]},"_transform_rotateZ_effect_hover_mobile_extra":{"unit":"deg","size":"","sizes":[]},"_transform_rotateZ_effect_hover_mobile":{"unit":"deg","size":"","sizes":[]},"_transform_rotate_3d_hover":"","_transform_rotateX_effect_hover":{"unit":"px","size":"","sizes":[]},"_transform_rotateX_effect_hover_tablet":{"unit":"deg","size":"","sizes":[]},"_transform_rotateX_effect_hover_mobile_extra":{"unit":"deg","size":"","sizes":[]},"_transform_rotateX_effect_hover_mobile":{"unit":"deg","size":"","sizes":[]},"_transform_rotateY_effect_hover":{"unit":"px","size":"","sizes":[]},"_transform_rotateY_effect_hover_tablet":{"unit":"deg","size":"","sizes":[]},"_transform_rotateY_effect_hover_mobile_extra":{"unit":"deg","size":"","sizes":[]},"_transform_rotateY_effect_hover_mobile":{"unit":"deg","size":"","sizes":[]},"_transform_perspective_effect_hover":{"unit":"px","size":"","sizes":[]},"_transform_perspective_effect_hover_tablet":{"unit":"px","size":"","sizes":[]},"_transform_perspective_effect_hover_mobile_extra":{"unit":"px","size":"","sizes":[]},"_transform_perspective_effect_hover_mobile":{"unit":"px","size":"","sizes":[]},"_transform_translate_popover_hover":"","_transform_translateX_effect_hover":{"unit":"px","size":"","sizes":[]},"_transform_translateX_effect_hover_tablet":{"unit":"px","size":"","sizes":[]},"_transform_translateX_effect_hover_mobile_extra":{"unit":"px","size":"","sizes":[]},"_transform_translateX_effect_hover_mobile":{"unit":"px","size":"","sizes":[]},"_transform_translateY_effect_hover":{"unit":"px","size":"","sizes":[]},"_transform_translateY_effect_hover_tablet":{"unit":"px","size":"","sizes":[]},"_transform_translateY_effect_hover_mobile_extra":{"unit":"px","size":"","sizes":[]},"_transform_translateY_effect_hover_mobile":{"unit":"px","size":"","sizes":[]},"_transform_scale_popover_hover":"","_transform_keep_proportions_hover":"yes","_transform_scale_effect_hover":{"unit":"px","size":"","sizes":[]},"_transform_scale_effect_hover_tablet":{"unit":"px","size":"","sizes":[]},"_transform_scale_effect_hover_mobile_extra":{"unit":"px","size":"","sizes":[]},"_transform_scale_effect_hover_mobile":{"unit":"px","size":"","sizes":[]},"_transform_scaleX_effect_hover":{"unit":"px","size":"","sizes":[]},"_transform_scaleX_effect_hover_tablet":{"unit":"px","size":"","sizes":[]},"_transform_scaleX_effect_hover_mobile_extra":{"unit":"px","size":"","sizes":[]},"_transform_scaleX_effect_hover_mobile":{"unit":"px","size":"","sizes":[]},"_transform_scaleY_effect_hover":{"unit":"px","size":"","sizes":[]},"_transform_scaleY_effect_hover_tablet":{"unit":"px","size":"","sizes":[]},"_transform_scaleY_effect_hover_mobile_extra":{"unit":"px","size":"","sizes":[]},"_transform_scaleY_effect_hover_mobile":{"unit":"px","size":"","sizes":[]},"_transform_skew_popover_hover":"","_transform_skewX_effect_hover":{"unit":"px","size":"","sizes":[]},"_transform_skewX_effect_hover_tablet":{"unit":"deg","size":"","sizes":[]},"_transform_skewX_effect_hover_mobile_extra":{"unit":"deg","size":"","sizes":[]},"_transform_skewX_effect_hover_mobile":{"unit":"deg","size":"","sizes":[]},"_transform_skewY_effect_hover":{"unit":"px","size":"","sizes":[]},"_transform_skewY_effect_hover_tablet":{"unit":"deg","size":"","sizes":[]},"_transform_skewY_effect_hover_mobile_extra":{"unit":"deg","size":"","sizes":[]},"_transform_skewY_effect_hover_mobile":{"unit":"deg","size":"","sizes":[]},"_transform_flipX_effect_hover":"","_transform_flipY_effect_hover":"","_transform_transition_hover":{"unit":"px","size":"","sizes":[]},"motion_fx_transform_x_anchor_point":"","motion_fx_transform_x_anchor_point_tablet":"","motion_fx_transform_x_anchor_point_mobile_extra":"","motion_fx_transform_x_anchor_point_mobile":"","motion_fx_transform_y_anchor_point":"","motion_fx_transform_y_anchor_point_tablet":"","motion_fx_transform_y_anchor_point_mobile_extra":"","motion_fx_transform_y_anchor_point_mobile":"","_background_background":"","_background_color":"","_background_color_stop":{"unit":"%","size":0,"sizes":[]},"_background_color_stop_tablet":{"unit":"%"},"_background_color_stop_mobile_extra":{"unit":"%"},"_background_color_stop_mobile":{"unit":"%"},"_background_color_b":"#f2295b","_background_color_b_stop":{"unit":"%","size":100,"sizes":[]},"_background_color_b_stop_tablet":{"unit":"%"},"_background_color_b_stop_mobile_extra":{"unit":"%"},"_background_color_b_stop_mobile":{"unit":"%"},"_background_gradient_type":"linear","_background_gradient_angle":{"unit":"deg","size":180,"sizes":[]},"_background_gradient_angle_tablet":{"unit":"deg"},"_background_gradient_angle_mobile_extra":{"unit":"deg"},"_background_gradient_angle_mobile":{"unit":"deg"},"_background_gradient_position":"center center","_background_gradient_position_tablet":"","_background_gradient_position_mobile_extra":"","_background_gradient_position_mobile":"","_background_image":{"url":"","id":"","size":""},"_background_image_tablet":{"url":"","id":"","size":""},"_background_image_mobile_extra":{"url":"","id":"","size":""},"_background_image_mobile":{"url":"","id":"","size":""},"_background_position":"","_background_position_tablet":"","_background_position_mobile_extra":"","_background_position_mobile":"","_background_xpos":{"unit":"px","size":0,"sizes":[]},"_background_xpos_tablet":{"unit":"px","size":0,"sizes":[]},"_background_xpos_mobile_extra":{"unit":"px","size":"","sizes":[]},"_background_xpos_mobile":{"unit":"px","size":0,"sizes":[]},"_background_ypos":{"unit":"px","size":0,"sizes":[]},"_background_ypos_tablet":{"unit":"px","size":0,"sizes":[]},"_background_ypos_mobile_extra":{"unit":"px","size":"","sizes":[]},"_background_ypos_mobile":{"unit":"px","size":0,"sizes":[]},"_background_attachment":"","_background_repeat":"","_background_repeat_tablet":"","_background_repeat_mobile_extra":"","_background_repeat_mobile":"","_background_size":"","_background_size_tablet":"","_background_size_mobile_extra":"","_background_size_mobile":"","_background_bg_width":{"unit":"%","size":100,"sizes":[]},"_background_bg_width_tablet":{"unit":"px","size":"","sizes":[]},"_background_bg_width_mobile_extra":{"unit":"px","size":"","sizes":[]},"_background_bg_width_mobile":{"unit":"px","size":"","sizes":[]},"_background_video_link":"","_background_video_start":"","_background_video_end":"","_background_play_once":"","_background_play_on_mobile":"","_background_privacy_mode":"","_background_video_fallback":{"url":"","id":"","size":""},"_background_slideshow_gallery":[],"_background_slideshow_loop":"yes","_background_slideshow_slide_duration":5000,"_background_slideshow_slide_transition":"fade","_background_slideshow_transition_duration":500,"_background_slideshow_background_size":"","_background_slideshow_background_size_tablet":"","_background_slideshow_background_size_mobile_extra":"","_background_slideshow_background_size_mobile":"","_background_slideshow_background_position":"","_background_slideshow_background_position_tablet":"","_background_slideshow_background_position_mobile_extra":"","_background_slideshow_background_position_mobile":"","_background_slideshow_lazyload":"","_background_slideshow_ken_burns":"","_background_slideshow_ken_burns_zoom_direction":"in","_background_hover_background":"","_background_hover_color":"","_background_hover_color_stop":{"unit":"%","size":0,"sizes":[]},"_background_hover_color_stop_tablet":{"unit":"%"},"_background_hover_color_stop_mobile_extra":{"unit":"%"},"_background_hover_color_stop_mobile":{"unit":"%"},"_background_hover_color_b":"#f2295b","_background_hover_color_b_stop":{"unit":"%","size":100,"sizes":[]},"_background_hover_color_b_stop_tablet":{"unit":"%"},"_background_hover_color_b_stop_mobile_extra":{"unit":"%"},"_background_hover_color_b_stop_mobile":{"unit":"%"},"_background_hover_gradient_type":"linear","_background_hover_gradient_angle":{"unit":"deg","size":180,"sizes":[]},"_background_hover_gradient_angle_tablet":{"unit":"deg"},"_background_hover_gradient_angle_mobile_extra":{"unit":"deg"},"_background_hover_gradient_angle_mobile":{"unit":"deg"},"_background_hover_gradient_position":"center center","_background_hover_gradient_position_tablet":"","_background_hover_gradient_position_mobile_extra":"","_background_hover_gradient_position_mobile":"","_background_hover_image":{"url":"","id":"","size":""},"_background_hover_image_tablet":{"url":"","id":"","size":""},"_background_hover_image_mobile_extra":{"url":"","id":"","size":""},"_background_hover_image_mobile":{"url":"","id":"","size":""},"_background_hover_position":"","_background_hover_position_tablet":"","_background_hover_position_mobile_extra":"","_background_hover_position_mobile":"","_background_hover_xpos":{"unit":"px","size":0,"sizes":[]},"_background_hover_xpos_tablet":{"unit":"px","size":0,"sizes":[]},"_background_hover_xpos_mobile_extra":{"unit":"px","size":"","sizes":[]},"_background_hover_xpos_mobile":{"unit":"px","size":0,"sizes":[]},"_background_hover_ypos":{"unit":"px","size":0,"sizes":[]},"_background_hover_ypos_tablet":{"unit":"px","size":0,"sizes":[]},"_background_hover_ypos_mobile_extra":{"unit":"px","size":"","sizes":[]},"_background_hover_ypos_mobile":{"unit":"px","size":0,"sizes":[]},"_background_hover_attachment":"","_background_hover_repeat":"","_background_hover_repeat_tablet":"","_background_hover_repeat_mobile_extra":"","_background_hover_repeat_mobile":"","_background_hover_size":"","_background_hover_size_tablet":"","_background_hover_size_mobile_extra":"","_background_hover_size_mobile":"","_background_hover_bg_width":{"unit":"%","size":100,"sizes":[]},"_background_hover_bg_width_tablet":{"unit":"px","size":"","sizes":[]},"_background_hover_bg_width_mobile_extra":{"unit":"px","size":"","sizes":[]},"_background_hover_bg_width_mobile":{"unit":"px","size":"","sizes":[]},"_background_hover_video_link":"","_background_hover_video_start":"","_background_hover_video_end":"","_background_hover_play_once":"","_background_hover_play_on_mobile":"","_background_hover_privacy_mode":"","_background_hover_video_fallback":{"url":"","id":"","size":""},"_background_hover_slideshow_gallery":[],"_background_hover_slideshow_loop":"yes","_background_hover_slideshow_slide_duration":5000,"_background_hover_slideshow_slide_transition":"fade","_background_hover_slideshow_transition_duration":500,"_background_hover_slideshow_background_size":"","_background_hover_slideshow_background_size_tablet":"","_background_hover_slideshow_background_size_mobile_extra":"","_background_hover_slideshow_background_size_mobile":"","_background_hover_slideshow_background_position":"","_background_hover_slideshow_background_position_tablet":"","_background_hover_slideshow_background_position_mobile_extra":"","_background_hover_slideshow_background_position_mobile":"","_background_hover_slideshow_lazyload":"","_background_hover_slideshow_ken_burns":"","_background_hover_slideshow_ken_burns_zoom_direction":"in","_background_hover_transition":{"unit":"px","size":"","sizes":[]},"_border_border":"","_border_width":{"unit":"px","top":"","right":"","bottom":"","left":"","isLinked":true},"_border_width_tablet":{"unit":"px","top":"","right":"","bottom":"","left":"","isLinked":true},"_border_width_mobile_extra":{"unit":"px","top":"","right":"","bottom":"","left":"","isLinked":true},"_border_width_mobile":{"unit":"px","top":"","right":"","bottom":"","left":"","isLinked":true},"_border_color":"","_border_radius":{"unit":"px","top":"","right":"","bottom":"","left":"","isLinked":true},"_border_radius_tablet":{"unit":"px","top":"","right":"","bottom":"","left":"","isLinked":true},"_border_radius_mobile_extra":{"unit":"px","top":"","right":"","bottom":"","left":"","isLinked":true},"_border_radius_mobile":{"unit":"px","top":"","right":"","bottom":"","left":"","isLinked":true},"_box_shadow_box_shadow_type":"","_box_shadow_box_shadow":{"horizontal":0,"vertical":0,"blur":10,"spread":0,"color":"rgba(0,0,0,0.5)"},"_box_shadow_box_shadow_position":" ","_border_hover_border":"","_border_hover_width":{"unit":"px","top":"","right":"","bottom":"","left":"","isLinked":true},"_border_hover_width_tablet":{"unit":"px","top":"","right":"","bottom":"","left":"","isLinked":true},"_border_hover_width_mobile_extra":{"unit":"px","top":"","right":"","bottom":"","left":"","isLinked":true},"_border_hover_width_mobile":{"unit":"px","top":"","right":"","bottom":"","left":"","isLinked":true},"_border_hover_color":"","_border_radius_hover":{"unit":"px","top":"","right":"","bottom":"","left":"","isLinked":true},"_border_radius_hover_tablet":{"unit":"px","top":"","right":"","bottom":"","left":"","isLinked":true},"_border_radius_hover_mobile_extra":{"unit":"px","top":"","right":"","bottom":"","left":"","isLinked":true},"_border_radius_hover_mobile":{"unit":"px","top":"","right":"","bottom":"","left":"","isLinked":true},"_box_shadow_hover_box_shadow_type":"","_box_shadow_hover_box_shadow":{"horizontal":0,"vertical":0,"blur":10,"spread":0,"color":"rgba(0,0,0,0.5)"},"_box_shadow_hover_box_shadow_position":" ","_border_hover_transition":{"unit":"px","size":"","sizes":[]},"_mask_switch":"","_mask_shape":"circle","_mask_image":{"url":"","id":"","size":""},"_mask_notice":"","_mask_size":"contain","_mask_size_tablet":"","_mask_size_mobile_extra":"","_mask_size_mobile":"","_mask_size_scale":{"unit":"%","size":100,"sizes":[]},"_mask_size_scale_tablet":{"unit":"px","size":"","sizes":[]},"_mask_size_scale_mobile_extra":{"unit":"px","size":"","sizes":[]},"_mask_size_scale_mobile":{"unit":"px","size":"","sizes":[]},"_mask_position":"center center","_mask_position_tablet":"","_mask_position_mobile_extra":"","_mask_position_mobile":"","_mask_position_x":{"unit":"%","size":0,"sizes":[]},"_mask_position_x_tablet":{"unit":"px","size":"","sizes":[]},"_mask_position_x_mobile_extra":{"unit":"px","size":"","sizes":[]},"_mask_position_x_mobile":{"unit":"px","size":"","sizes":[]},"_mask_position_y":{"unit":"%","size":0,"sizes":[]},"_mask_position_y_tablet":{"unit":"px","size":"","sizes":[]},"_mask_position_y_mobile_extra":{"unit":"px","size":"","sizes":[]},"_mask_position_y_mobile":{"unit":"px","size":"","sizes":[]},"_mask_repeat":"no-repeat","_mask_repeat_tablet":"","_mask_repeat_mobile_extra":"","_mask_repeat_mobile":"","hide_desktop":"","hide_tablet":"","hide_mobile_extra":"","hide_mobile":"","custom_css":"","wpr_custom_css":""},"defaultEditSettings":{"defaultEditRoute":"content"},"elements":[],"widgetType":"heading","htmlCache":"\t\t
IMAT 2024 File (only Pictures) – Download Now
\t\t\t\t
\n\t\t","editSettings":{"defaultEditRoute":"content"}}],"editSettings":{"defaultEditRoute":"layout","panel":{"activeTab":"layout","activeSection":"section_layout_container"}},"htmlCache":null}]}